PW Only IAS Prelims 2024 Test 3 With Solution

You might also like

Download as pdf or txt
Download as pdf or txt
You are on page 1of 44

https://upscpdf.com/ https://upscpdf.

com/

Prelims Wallah
Test
Test-3 of 38
(Ancient, Medieval and Art and Culture NCERT)

TARGET 2024

SCAN HERE FOR TEST SERIES ACCESS

https://upscpdf.com/ https://upscpdf.com/
https://upscpdf.com/ https://upscpdf.com/

Prelims Wallah Test 2024


Test 3: Ancient, Medieval and Art & Culture NCERT Test
1. Consider the following: 4. With reference to the Vijayanagar Empire of
1. Rank medieval India, consider the following pairs :
2. Salary Term Associated rulers
3. Military 1. Gajapati : Rulers of Orissa
Which of the above were fixed in the 2. Ashvapati : Rayas of Vijayanagara
mansabdari system used by the Mughals? 3. Narapati : Sultans of Deccan
(a) 1 only How many of the above pairs are correctly
(b) 2 only matched?
(c) 1 and 2 only (a) Only one pair
(d) 1, 2 and 3 (b) Only two pairs
(c) All three pairs
2. Consider the following statements with
(d) None of the pairs
reference to the term “Hindustan” used in the
early medieval times:
5. Consider the following:
1. Minhaj-i-Siraj used it in context of the
1. Rotation of Earth
social structure present in the country at
2. Eclipses
that time.
3. Calculation of Circumference of a Circle
2. Babur used Hindustan to describe the
4. Elaborate Surgical procedure
geography, the fauna, and the culture of
How many of the above are mentioned in the
the inhabitants of the subcontinent.
‘Aryabhatiyam’, a book written by
Which of the statements given above is/are
Aryabhata?
correct?
(a) Only one
(a) 1 only
(b) Only two
(b) 2 only
(c) Only three
(c) Both 1 and 2
(d) All four
(d) Neither 1 nor 2

6. With reference to the mansabdars who served


3. Aryabhata’s work included which of the
under the Mughals, consider the following
following subjects?
statements:
1. Astrology
1. All mansabdars directly administered
2. Astronomy
and collected revenue from their jagirs.
3. Mathematics
2. They only had rights to the revenue from
Select the correct answer using the codes
their assignments, usually collected by
given below:
their servants.
(a) 1 and 2 only
3. They had the responsibility of
(b) 1 and 3 only maintaining a specified number of
(c) 2 and 3 only sawars and getting them registered.
(d) 1, 2 and 3

https://upscpdf.com/ https://upscpdf.com/
https://upscpdf.com/ https://upscpdf.com/

Which of the statements given above is/are


correct? 9. Match the following correctly and select the
(a) 1 only correct answer from the codes given below:
(b) 2 only Epics Authors
(c) 1 and 3 only
A. Silappadikaram 1. Valmiki
(d) 2 and 3 only
B. Manimekalai 2. Sattanar
7. With respect to Hyderabad, consider the C. Meghaduta 3. Kalidasa
following statements: D. Ramayana 4. Ilango
1. Nizam-ul-Mulk Asaf Jah was an Code:
important member of the court of the (a) A-2; B-4; C-3; D-1
Mughal Emperor Aurangzeb. (b) A-4; B-3; C-1; D-2
2. The state of Hyderabad was constantly (c) A-4; B-2; C-3; D-1
engaged in a struggle against the (d) A-2; B-1; C-3; D-4
Marathas.
Which of the statements given above is/are 10. Consider the following statements:
incorrect? 1. The Mughal mansabdari system used a
(a) 1 only numerical value called zat to determine
(b) 2 only rank and salary.
(c) Both 1 and 2 2. The nobles with zat of 1,000 ranked
(d) Neither 1 nor 2 higher than those with zat of 5,000.
Which of the statements given above is/are
8. Consider the following statements regarding correct?
the Vijayanagara Empire: (a) 1 only
Statement-I: The import of high quality (b) 2 only
horses from Arabia and Central Asia was (c) Both 1 and 2
very important. (d) Neither 1 nor 2
Statement-II: The warfare during these times
depended upon effective cavalry. 11. Consider the following statements with
Which one of the following is correct with reference to the History of Indian coinage:
respect to the above statements? 1. Kushanas issued many gold as well as
(a) Both Statement-I and Statement-II are copper coins.
correct and Statement-II is the correct 2. The reverse of the Gupta coins has
explanation for Statement-I religious symbols.
(b) Both Statement-I and Statement-II are 3. The rulers of Satavahanas banned the
correct and Statement-II is not the correct circulation of Punch-marked coins.
explanation for Statement-I Which of the statements given above is/are
(c) Statement-I is correct but Statement-II is correct?
incorrect (a) 1 only
(d) Statement-I is incorrect but Statement-II (b) 1 and 2 only
is correct (c) 2 and 3 only
(d) 1, 2 and 3

https://upscpdf.com/ https://upscpdf.com/
https://upscpdf.com/ https://upscpdf.com/

12. ‘Harmika’, and 'Yashti' are the terms related 16. Which of the following statements is
to which of the following? incorrect in the context of ancient Indian
(a) Bodhisattva languages and scripts?
(b) Jainism literature 1. The Kharoshthi script was written from
(c) Jain scholars right to left.
(d) Buddhist architecture 2. The Brahmi script was deciphered by
James Prinsep.
13. With reference to the prominent medieval 3. Brahmi was the main script until the end
literary work ‘Lilatilakam’, consider the of Gupta times.
following statements: Select the correct answer using the code
1. It deals with grammar and poetics. given below:
2. It was written in Manipravalam (a) 1 and 2 only
Which of the statements given above is/are (b) 2 and 3 only
correct? (c) 3 only
(a) 1 only (d) None of the above
(b) 2 only
(c) Both 1 and 2 17. With reference to “jatis” in the early
(d) Neither 1 nor 2 Medieval history of India, consider the
following statements:
14. Consider the following pairs: 1. Rules and regulations to be followed by
Books Author them were elucidated in Rigveda.
1. Silappadikaram Sattanar 2. The rules and regulations were enforced
2. Manimekalai Valayapathi by a group of elders called jati panchayat.
3. Meghaduta Kalidas 3. No other rules had to be followed by
How many of the above pairs are correctly them.
matched? How many of the above statements are
(a) Only one pair correct?
(b) Only two pairs (a) Only one
(c) All three pairs (b) Only two
(d) None of the pairs (c) All three
(d) None
15. Which of the following terms was used for
the revenue system introduced by Akbar, that 18. Who among the following Gupta rulers is
relied on careful surveys on crop yields, depicted on coins playing a vina?
areas, and prices and fixed taxes in cash for (a) Chandragupta II
each crop based on these surveys? (b) Samudragupta
(a) Mansabdari (c) Sri Gupta
(b) Iqta (d) Vishnugupta
(c) Zabt
(d) Zamindari

https://upscpdf.com/ https://upscpdf.com/
https://upscpdf.com/ https://upscpdf.com/

19. With reference to the zabt revenue system 22. Who among the following Mughal rulers
introduced in the reign of Mughal Emperor minted silver coins with their titles on one
Akbar, consider the following statements: side and the inscription "struck in the name
1. Taxes were fixed on each crop in cash. of the Queen Begum" on the other side?
2. It was most successful in Gujarat and (a) Jahangir
Bengal. (b) Akbar
3. Provinces were subdivided into revenue (c) Shahjahan
circles, each with its own schedule of (d) Humayun
revenue rates.
How many of the above mentioned features 23. Consider the following pairs:
are correct? Philosophy Founded by
(a) Only one 1. Nyaya : Jaimini
(b) Only two 2. Yoga : Patanjali
(c) All three 3. Samkhya : Vyasa
(d) None 4. Vedanta : Kapila
How many of the pairs given above are
20. The followers of the cult of Jagannatha were correct?
devotees of: (a) Only one pair
(a) Shiva (b) Only two pairs
(b) Vishnu (c) Only three pairs
(d) All four pairs
(c) Sarabha
(d) Brahma
24. In the context of medieval history, consider
the following statements about the
21. ‘The inscription recounts how a governor
Rashtrakutas:
under the Mauryas built a lake with
1. Their empire was confined mainly to the
embankments and channels. A fierce storm
Malwa region.
later wrecked the lake, but Rudradaman,
2. Initially, they were subordinate to the
ruling the area, financed its restoration
Chalukyas of Karnataka.
without taxing his people.’
3. Dantidurga overthrew the last Chalukya
Identify the inscription from the options
King.
given below:
Which of the statements given above is/are
(a) Allahabad inscription
correct?
(b) Hathigumpha inscription
(a) 1 and 2 only
(c) Girnar inscription
(d) Udaygiri inscription (b) 2 and 3 only
(c) 1, 2 and 3
(d) 3 only

https://upscpdf.com/ https://upscpdf.com/
https://upscpdf.com/ https://upscpdf.com/

25. With reference to the ancient history of India, How many of the above pairs are incorrectly
the term ‘sarthavaha’ refers to: matched?
(a) Chief banker of the city (a) Only one
(b) Chief craftsman (b) Only two
(c) Leader of the merchant caravans
(c) All three
(d) Chief judicial officer
(d) None
26. Consider the following statements:
29. Consider the following statements:
1. Workers will be paid according to
1. A Bodhisattva is a compassionate person
quantity and quality of work.
walking the path to enlightenment.
2. All women must get the raw materials
2. A Bodhisattva postpones their own
themselves from the superintendent.
salvation to aid all living beings in
Which of the above statements about the
reaching their salvation.
rules of spinning and weaving are mentioned
3. The concept of a Bodhisattva is an
in Kautilya’s Arthashastra?
important aspect of Hinayana Buddhism.
(a) 1 only
Which of the statements given above
(b) 2 only
characterises Bodhisattva?
(c) Both 1 and 2
(d) Neither 1 nor 2 (a) 1 and 2 only
(b) 1 and 3 only
27. With reference to Mughal India, consider the (c) 2 and 3 only
following pairs: (d) 1, 2 and 3
List 1 List 2
1. Umara : High Noble 30. Consider the following statements:
2. Faujdar : Revenue Farmer Statement-I: Mughals, Marathas, and English
3. Subadar : Provincial Governor East India Company attempted to gain
4. Chauth : Tax collected by control over the Jagannatha temple.
Marathas Statement-II: Control over the temple would
Which of the pairs given above is/are help rulers make their rule acceptable to the
correctly matched? local people.
(a) 4 only Which one of the following is correct with
(b) 1, 2 and 4 only respect to the above statements?
(c) 1, 3 and 4 only (a) Both Statement-I and Statement-II are
(d) 3 and 4 only correct and Statement-II is the correct
explanation for Statement-I
28. With reference to social organisation in the (b) Both Statement-I and Statement-II are
Sangam Age, consider the following pairs: correct and Statement-II is not the correct
Word Meaning explanation for Statement-I
1. Vellalar : Village headman (c) Statement-I is correct but Statement-II is
2. Uzhavar : Ordinary ploughmen incorrect
3. Adimai : Slaves (d) Statement-I is incorrect but Statement-II
is correct

https://upscpdf.com/ https://upscpdf.com/
https://upscpdf.com/ https://upscpdf.com/

31. Which of the following centres took over the 35. In the context of medieval Indian history, the
role of the capital of Magadha in the fourth term ‘Vetti’, refers to:
century BCE, succeeding Rajagaha? (a) The coin system of the kushana period
(a) Ujjain (b) The tax taken not in cash but in the form
(b) Vaishali of forced labour
(c) Pataliputra (c) The land grant to Bhrahmanas
(d) Taxila (d) The form of land revenue

32. Consider the following statements: 36. “This city is located at the crossroads of two
1. The entire settlement of Dholavira was major routes of travel and trade—from the
fortified. northwest to the east and from north to south.
2. The Great Bath was surrounded by walls It is also a religious centre of Hinduism,
on three sides and steps on the fourth Jainism, and Buddhism, and it became the
side. second capital of the Kushan Kingdom”.
3. In the Indus Valley Civilization, Harappa Which of the following cities is best
was the first site to be discovered. described by the paragraph given above?
Which of the above statements is/are correct (a) Mathura
regarding the Indus Valley Civilization? (b) Varanasi
(a) 1 and 2 only (c) Ujjain
(b) 2 and 3 only (d) Kanchipuram
(c) 1 and 3 only
(d) 1, 2, and 3 37. With reference to Sikhism in medieval India,
consider the following pairs :
33. The famous Koh-i-nur / Kohinoor diamond List I List II
was taken away from India in 18th century 1. Jathas : Bands (groups)
during invasion of Delhi by: 2. Dal Khalsa : Grand Army of Sikhs
(a) Ahmad Shah Abdali 3. Gurmatas : Resolutions of the Guru
(b) Nadir Shah 4. Rakhi : System offering
(c) Nizam-ul-Mulk protection to cultivators
(d) Saadat Khan
Which of the pairs given above is/are
34. Consider the following scripts: incorrectly matched?
1. Aramaic (a) 4 only
2. Kharosthi (b) 2 and 4 only
3. Brahmi (c) 1 and 3 only
4. Cirth (d) None of the above
How many of the above scripts were used in
the Asokan inscriptions? 38. Who among the following was the court poet
(a) Only one of ‘Samudragupta’?
(b) Only two (a) Kalidasa
(c) Only three (b) Harishena
(d) All four (c) Banabhatta
(d) Vishakhadatta

https://upscpdf.com/ https://upscpdf.com/
https://upscpdf.com/ https://upscpdf.com/

39. The Jagannath Temple in Puri, was 43. Consider the following statements :
constructed during the reign of which 1. The term 'India' has been derived from
dynasty? the Greek word 'Indos' which denoted the
(a) Maurya dynasty land to the east of the river Indus.
(b) Chola dynasty 2. The word 'Bharata' is found in the
historical texts only after the
(c) Ganga dynasty
Mahabharata period.
(d) Gupta dynasty
Which of the statements given above is/are
correct?
40. With reference to the history of coins in
(a) 1 only
India, consider the following statements:
(b) 2 only
1. The first gold coins were issued in the (c) Both 1 and 2
first century CE by the Kushans. (d) Neither 1 nor 2
2. The first coins that can be attributed to
kings were issued by the Indo-Greeks. 44. The wall relief showing Vishnu as
3. Tribal republics did not issue coins. Narasimha, from Ellora Caves was
Which of the statements given above are commissioned during the reign of which of
correct? the following kingdoms?
(a) 1 and 2 only (a) Chalukya
(b) 2 only (b) Chandela
(c) 2 and 3 only (c) Rashtrakuta
(d) Vijayanagara
(d) 1, 2 and 3
45. Which of the following cities were the
41. Which of the following Buddhist text alludes
important provincial centres of the Mauryan
to a conversation between King Ajatasattu,
empire?
and the Buddha?
1. Taxila
(a) Tibetan Book of the Dead (Bardo
2. Ujjayini
Thodol)
3. Tosali
(b) Sutta Pitaka
4. Suvarnagiri
(c) Jataka Tales
Select the correct answer using the code
(d) Mahayana Sutras
given below:

42. Sawai Jai Singh is known to have constructed (a) 1 and 2 only
five astronomical observatories commonly (b) 2, 3 and 4 only
known as Jantar Mantar. Which of the (c) 1 and 4 only
following cities does not have any of these (d) 1, 2, 3 and 4
observatories?
(a) Ujjain
(b) Mathura
(c) Udaipur
(d) Varanasi

https://upscpdf.com/ https://upscpdf.com/
https://upscpdf.com/ https://upscpdf.com/

46. In the context of Jats, consider the following How many of the above pairs are incorrectly
statements: matched?
1. Traditionally, they were agricultural (a) Only one pair
communities in southern India. (b) Only two pairs
2. Suraj Mal was one of their prominent (c) All three pairs
leaders. (d) None of the pairs
Which of the statements given above is/are 50. In the context of Harappan civilization,
incorrect ? consider the following statements:
(a) 1 only 1. Evidence of ploughed fields was found at
(b) 2 only Kalibangan
(c) Both 1 and 2 2. Bull was known to the people of Indus
(d) Neither 1 nor 2 valley civilization
3. Irrigation was used for growing crops.
47. With reference to the cultural history of India, How many of the above statements is/are
consider the following statements: correct?
1. Puranas were written in simple Sanskrit (a) Only one
and meant to be read aloud as stories of (b) Only two
deities. (c) All three
2. Vaishnavism proclaimed that there are (d) None
primarily one hundred avatars of Vishnu.
3. Shaivism symbolized Shiva as a human 51. Consider the following pairs:
figure as well as a Linga. Personality Literary Work
Which of the statements given above are 1. Surdasa : Bijak
correct? 2. Jayadeva : Gita Govinda
(a) 1 and 2 only 3. Kabir : Sahitya Lahari
(b) 1 and 3 only 4. Malik Mohammad : Padmavat
Jayasi
(c) 2 and 3 only
Which of the pairs given above are correctly
(d) 1, 2 and 3
matched?
(a) 1 and 3 only
48. Which of the following Buddhist texts
(b) 2 and 4 only
provides an insight into women’s social and
(c) 3 and 4 only
spiritual experiences?
(d) 1, 2 and 4 only
(a) Cīvaka Cintāmaṇi
(b) Therigatha
52. With reference to ‘Gupta period’, consider
(c) Kuṇṭalakēci
the following statements:
(d) Valayapathi
1. The Guptas entered into matrimonial
alliances with the Vakataka dynasty of
49. Consider the following pairs:
the Deccan.
Tribe Region of Influence
2. Women typically did not have the
1. Gakkhars : Punjab autonomy to independently access
2. Ahoms : North-East resources such as land.
3. Arghuns : Sindh

https://upscpdf.com/ https://upscpdf.com/
https://upscpdf.com/ https://upscpdf.com/

3. Some of the land grants were exempted 55. Consider the following statements about
from paying land revenue and other dues Ashoka’s dhamma:
to the king. 1. Ashoka’s dhamma promotes the
Which of the statements given above are performance of various sacrifices and
correct? rituals.
(a) 1 and 2 only 2. Dhamma mahamatta were the officials
(b) 2 and 3 only appointed to teach people about dhamma.
(c) 1 and 3 only 3. Ashoka sent messengers to spread ideas
(d) 1, 2 and 3 about dhamma to Syria, Egypt and Sri
Lanka.
53. Consider the following statements with How many of the above statements are
reference to the Sangam age of Indian incorrect?
History: (a) Only one
1. Punch-marked coins were inscribed (b) Only two
coins found over most of Indian (c) All three
Subcontinent in the early times. (d) None
2. Dasa Karmakara did not own land and
worked on fields owned by others. 56. With reference to the medieval history of
Which of the statements given above is/are India, the term Malfuzats, Maktubat and
correct? Tazkiras were related to which one of the
(a) 1 only following?
(b) 2 only (a) Converted muslims
(c) Both 1 and 2 (b) Islamic prayers
(d) Neither 1 nor 2 (c) Land grants
(d) Sufi texts
54. With reference to medieval India, consider
the following pairs: 57. Consider the following statements about the
Dynasty Present State rajas of Rigveda:
1. Gurjara-Pratiharas : Madhya Pradesh 1. They had well-fortified and thriving
2. Rashtrakutas : West Bengal capitals.
3. Palas : Karnataka 2. They owned their armies.
4. Cheras : Kerala
3. They did not have centralised taxation.
How many of the above pairs are correctly
4. Sons did not automatically succeed
matched?
fathers as rajas.
(a) Only one
How many of the statements given above are
(b) Only two
correct?
(c) Only three
(a) Only one
(d) All four
(b) Only two
(c) Only three
(d) All four

https://upscpdf.com/ https://upscpdf.com/
https://upscpdf.com/ https://upscpdf.com/

10

58. Consider the following statements about the 61. With reference to the Mauryan Empire,
‘Gond’ tribes in the medieval period in India: consider the following statements:
1. They practised shifting cultivation. 1. Samharta during the Mauryan Age was
2. The tribe was divided into many smaller the highest officer of tax assessment.
clans, and each clan had its own raja. 2. Megasthenes mentions six boards of five
members each to control the six wings of
3. Gond kingdom is mentioned in the Akbar
the military under the Mauryan empire.
Nama.
3. Chandragupta with the assistance of
How many of the above statements are
Kautilya overthrew the Nandas.
correct?
How many of the above statements are
(a) Only one
incorrect?
(b) Only two
(a) Only one
(c) All three
(b) Only two
(d) None
(c) All three
(d) None
59. “At this site, a structure was found across a
lane to the north of which lay a smaller
62. Consider the following pairs:
building with eight bathrooms, four on each
Traveller Country of Origin
side of a corridor. This structure was also
1. Al-Biruni Uzbekistan
used for ritual bathing and was surrounded by
2. Ibn Battuta Morocco
a corridor on all four sides”.
3. Francois Bernier France
The structure described in the above
How many of the pairs above are correctly
paragraph is found at which of the following
matched?
Harappan sites?
(a) Only one
(a) Dholavira
(b) Only two
(b) Mohenjodaro
(c) All three
(c) Kalibangan
(d) None
(d) Harappa

63. With regard to reasons behind emergence of


60. Consider the following statements with
Magadha as the most powerful
reference to Sikhism:
Mahajanapada, consider the following
1. Guru Nanak rejected the scriptures of
statements:
both Hindus and Muslims.
2. Guru Angad was the successor of Guru 1. Ambitious kings and ministers such as
Nanak. Bimbisara, Ajatasatru, and Mahapadma
3. The Adi Granth Sahib was compiled by Nanda.
Guru Arjan 2. Presence of iron mines which provided
Which of the statements given above is/are resources for tools and weapons.
correct? 3. Elephants which were an important
(a) 1 and 2 only component of the warfare techniques in
(b) 2 and 3 only the era were found easily in the nearby
(c) 1 and 3 only forests.
(d) 1, 2 and 3

https://upscpdf.com/ https://upscpdf.com/
https://upscpdf.com/ https://upscpdf.com/

11

How many of the above statements are (d) None


incorrect? 66. Consider the following events in ancient
(a) Only one India:
(b) Only two 1. End of the Gana or Sanghas, different
(c) All three forms of government in Ancient India
(d) None 2. Rise of Mahajanapadas
3. Composition of the Digha Nikaya
64. With reference to ancient history and Arrange the above events in chronological
tradition of India, consider the following order from past to present:
pairs: (a) 2-3-1
List 1 List 2 (b) 1-3-2
1. Kutagarashala refers to, a hut with (c) 1-2-3
pointed roof, where
(d) 3-2-1
debates were held
2. Rajasuya and refer to elaborate
67. Consider the following statements:
Ashvamedha sacrifices and rituals
1. The Kushana rulers incorporated the title
performed by kings
"devaputra," which translates to ‘son of
3. Mahavira and Known for
god’.
Buddha supporting the
2. The Kushanas issued gold coins that
hierarchy and
were almost identical in weight to Roman
authority of Vedas
coins.
Which of the statements given above is/are
Which of the pairs given above are correctly
correct?
matched?
(a) 1 only
(a) 1 and 2 only (b) 2 only
(b) 1 and 3 only (c) Both 1 and 2
(c) 2 and 3 only (d) Neither 1 nor 2
(d) 1, 2 and 3
68. Consider the following statements:
65. With reference to Chishti tradition, consider 1. Al Biruni arrived at Ghazni with Sultan
the following statements: Mahmud Ghazni after his attack on
1. The Chishti order was named after its Khwarizm.
founders. 2. The works of travel literature already
2. Salim Chishti was popularly known as present in Ghazni at the time of Al-Biruni
Gharib Nawaz. dealt with lands as far apart as the Sahara
3. The saints of Chishti tradition maintained desert in the west and the River Volga in
absolute distance from political power. the north.
How many of the above statements are Which of the statements given above is/are
incorrect? correct?
(a) Only one (a) 1 only
(b) Only two (b) 2 only
(c) All three (c) Both 1 and 2

https://upscpdf.com/ https://upscpdf.com/
https://upscpdf.com/ https://upscpdf.com/

12

(d) Neither 1 nor 2 73. Consider the following regions in ancient


69. Consider the following statements: India with respect to the presence of
1. Alvars and Nayannars in the ninth Megalithic structures:
century were drawn only from the upper 1. Deccan
castes. 2. South India
2. Early Bhakti saints were sharply critical
3. North-east
of Buddhists and Jainas.
4. Kashmir
3. Nalayira Divyaprabandham was known
How many of the options given above are
as Tamil Veda.
correct?
How many of the above statements are
correct? (a) Only one
(a) Only one (b) Only two
(b) Only two (c) Only three
(c) All three (d) All four
(d) None
74. Consider the following statements regarding
70. In the context of the Mauryan period, the Vedic literature:
term ‘Pativedakas’ refers to: 1. The term ‘Veda’ means ‘Knowledge/
(a) Poets in Ashoka's court Wisdom’.
(b) Officers reporting the affairs of the 2. The Atharva Veda primarily consists of
people to the king various details of rules to be observed at
(c) Officers in charge of propagation of the time of sacrifice.
Ashoka's Dhamma 3. The Brahmanas are the commentaries on
(d) Officers for the interpretations of law hymns and rituals.
How many of the above statements are
71. Who among the following was the publisher correct?
of ‘Bhilsa Topes’, one of the earliest works (a) Only one
on Sanchi? (b) Only two
(a) Alexander Cunningham (c) All three
(b) James Fergusson
(d) None
(c) Walter Elliot
(d) John Marshall
75. She was a princess from the Mewar region
who renounced her husband and recognized
72. In the context of the Ahom Kingdom
Krishna as her lover. She was a disciple of
of Medieval India, who among the following
Raidas. She was popular for her bhajans
were called paiks?
which were orally transmitted. This is the
(a) Tribal forced labourers most likely description of who among the
(b) Tribal deity following?
(c) Tribal farmers (a) Lal Ded
(d) Tribal ropemakers (b) Mirabai
(c) Gargi
(d) Gangasati

https://upscpdf.com/ https://upscpdf.com/
https://upscpdf.com/ https://upscpdf.com/

13

76. Consider the following pairs: 79. “ He was a French traveller and was a doctor,
Buddhist text Content political philosopher, and historian. He also
1. Vinaya Pitaka : Rules and served as a physician to Prince Dara Shukoh,
regulations for the eldest son of Emperor Shah Jahan”.
monastic order Which of the following travellers is correctly
2. Abhidhamma : Buddha’s described in the paragraph above?
Pitaka teachings (a) Jean Baptiste Tavernier
3. Sutta Pitaka : Philosophical (b) Francois Bernier
matters (c) Nicolo De Conti
How many of the above pairs are correctly (d) Marco Polo
matched?
(a) Only one pair 80. Consider the following statements with
(b) Only two pairs regard to the Marathas during the 18th
(c) All three pairs century:
(d) None of the pairs 1. The Maratha rulers had the support of
some powerful warrior families.
77. “He was a Moroccan traveller and became 2. The Marathas seized Malwa and Gujarat
the qazi of Delhi under Sultan Muhammad from the Mughals.
bin Tughlaq, and later on he went to China as Which of the statements given above is/are
the sultan’s envoy to the Mongol ruler.” correct ?
Who among the following travellers is best (a) 1 only
described by the paragraph above? (b) 2 only
(a) Ibn Battuta (c) Both 1 and 2
(b) Al-Biruni (d) Neither 1 nor 2
(c) Abdur Razzak
(d) Nicolo de conti 81. With reference to Nanda Dynasty, consider
the following statements:
78. Consider the following statements: 1. The foundation of the new capital of
1. In ancient India, manuscripts were Magadha at Patliputra was laid down by
written both on Palm leaf and Birch Mahapadama Nanda.
bark. 2. The Hathigumpha Inscription refers to an
2. Only Sanskrit was used for writing aqueduct built by King Nanda.
manuscripts in ancient India. 3. Alexander invaded India during the rule
3. Manuscripts dealt with both religious and of Nandas.
non-religious subjects, including Which of the statements given above are
literature. incorrect?
How many of the statements given above (a) 1 only
is/are correct? (b) 2 and 3 only
(a) Only one (c) 1 and 3 only
(b) Only two (d) None of the above
(c) All three
(d) None

https://upscpdf.com/ https://upscpdf.com/
https://upscpdf.com/ https://upscpdf.com/

14

82. With reference to Kathak dance, consider the 85. In the context of the Maratha economy,
following statements: consider the following statements:
1. Kathak dancers were originally a caste of 1. Burhanpur became an important trading
storytellers in temples of north India. centre under the Marathas.
2. Kathak evolved into a distinct mode of 2. Indore functioned as an important
dance during the bhakti movement. commercial centre under the Sindhia
3. Rajasthan gharana and Lucknow gharana rulers.
are the traditions of Kathak. Which of the statements given above is/are
How many of the above statements are incorrect ?
incorrect? (a) 1 only
(a) Only one (b) 2 only
(b) Only two (c) Both 1 and 2
(c) All three (d) Neither 1 nor 2
(d) None
86. Which among following terms best describes
83. Consider the following ancient sites: the land assignments given to some of the
1. Kandhar Rajput Rulers by the Mughals after their
2. Manshera defeat?
3. Kalsi (a) Dakhnis
4. Meerut (b) Mansab
How many of the places mentioned above (c) Watan
were the sites of major Ashokan rock edicts? (d) Zamindari
(a) Only one
(b) Only two 87. With reference to the ‘Jaina philosophy’,
(c) Only three consider the following statements:
(d) All four 1. The cycle of birth and rebirth is shaped
through karma.
84. With reference to archaeological sites and 2. Everything in the world is lifeless and
their locations, consider the following pairs: constantly changing.
Archaeological site Location 3. The moderation between penance and
1. Mahagara Chhattisgarh self-indulgence.
2. Burzahom Haryana Which of the statements given above is/are
3. Hallur Kerala correct?
How many of the above pairs are correct? (a) 1 and 3 only
(a) Only one pair (b) 1 only
(b) Only two pairs (c) 2 and 3 only
(c) All three pairs (d) 1, 2 and 3
(d) None of the pairs

https://upscpdf.com/ https://upscpdf.com/
https://upscpdf.com/ https://upscpdf.com/

15

88. With reference to medieval Indian History 91. Consider the following statements with
under Mughals, consider the following pairs: reference to Harappa:
Word Meaning 1. The longest Inscription found there
1. Pargana Administrative contains 26 signs and has not been
subdivision of deciphered yet.
Mughal province 2. The Harappan script was written from
2. Peshkash Tribute collected by left to right.
the Mughal state. 3. Seals and Sealings were used for long-
3. Muqaddam Panchayat head distance communication.
How many of the above pairs are correctly 4. A precise system of weights and
matched? measurements was used.
(a) Only one pair How many of the above statements are is/are
(b) Only two pairs correct?
(c) All three pairs (a) Only one
(d) None of the pairs (b) Only two
(c) Only three
89. In the context of the regional states of the (d) All four
eighteenth century India, which one of the
following best describes the term 92. With reference to the life of Gautama Buddha
‘Nayakas’? as described in the Buddhist literature,
(a) The independent Telugu warrior chiefs Consider the following places and their
of the Deccan plateau. significance in his life:
(b) The skilled soldiers brought by Asaf Jah Location Significance
from the northern India. 1. Lumbini Buddha’s
(c) The rulers of the kingdom of Awadh. birthplace.
(d) The local chiefs of the state of Mysore. 2. Bodh Gaya place where he
attained
90. Which of the following phrases defines the enlightenment.
land ownership system in the Mughal Empire 3. Saranath place of his first
as described by François Bernier? sermon.
(a) The land was held in collective 4. Kushinagara place where he
ownership by the nobility. attained Nibbana
(b) The Land ownership was in private How many pairs given above are correctly
hands, held by individual farmers. matched?
(c) The ownership of land was vested in (a) Only one pair
religious institutions. (b) Only two pairs
(d) The emperor possessed sole ownership
(c) Only three pairs
of all land and allocated it among nobles.
(d) All four pairs

https://upscpdf.com/ https://upscpdf.com/
https://upscpdf.com/ https://upscpdf.com/

16

93. Consider the following statements about the 96. With reference to Mughal India’s description
Miniature paintings in medieval history: by Francois Bernier, consider the following
1. These are the small-sized paintings, statements:
generally done in water colour on wall 1. Artisans lacked motivation to enhance
and rocks. the quality of their products.
2. The Mughal emperors Akbar, Jahangir 2. There was a thriving and prosperous
and Shah Jahan patronised highly skilled merchant community.
painters. 3. India exported substantial quantities of
Which of the statements given above is/are the world's precious metals.
correct? How many of the above statements is/are
correct?
(a) 1 only
(a) Only one
(b) 2 only
(b) Only two
(c) Both 1 and 2 (c) All three
(d) Neither 1 nor 2 (d) None
94. According to Portuguese traveller Domingo
Paes, the House of Victory referred to which 97. With reference to the cultural history of
of the following architectures of Hampi? India, which one of the following is the
1. The Mahanavami Dibba correct description of the term
2. The Lotus Temple ‘kutagarashala’?
3. The Audience hall (a) A merchant guild in ancient India
4. The Virupaksha Temple (b) A type of ancient script used in
Select the correct answer using the code inscriptions
given below. (c) Huts with pointed roofs
(d) The architectural style of cave temples
(a) 1 and 2 only
during the Gupta period
(b) 1 and 3 only
(c) 2 and 3 only 98. Which one of the following travellers
(d) 3 and 4 only described Vijaynagar as “the best-provided
city in the world”?
95. “It is located in the Rann of Kutch, and unlike
(a) Domingo Paes
other cities of Harappa, it was divided into
(b) Fernao Nuniz
three parts. Here, large letters of Harappan
(c) Abdur Razzaq
script made of white stone and inlaid into the
(d) Nicolo de Conti
wood were found. It also has a water
reservoir used perhaps for storing water for
agriculture.”
Which of the following Harappan sites is
appropriately described in the above
paragraph?
(a) Kalibangan
(b) Lothal
(c) Banawali
(d) Dholavira

https://upscpdf.com/ https://upscpdf.com/
https://upscpdf.com/ https://upscpdf.com/

17

99. With reference to Vijaynagar Architecture,


consider the following pairs :
Architecture type Most Probable
Purpose/characte
ristics
1. Lotus Mahal Served as Council
Chamber
2. Hazara Rama Temple shrine for
temples common people.
3. The Ceremonial and
Mahanavami administrative
Dibba purpose
4. The Vittala Famous for its
Temple stone chariot
How many of the above pairs are correctly
matched?
(a) Only one pair
(b) Only two pairs
(c) Only three pairs
(d) All four pairs

100. During the seventeenth century, the term


‘khud-kashta’ and ‘pahi-kashta’ were
associated with:
(a) Peasants
(b) Saint tradition
(c) Trible land
(d) Islamic sects

https://upscpdf.com/ https://upscpdf.com/
https://upscpdf.com/ https://upscpdf.com/

Prelims Wallah Test 2024


Test 3: Ancient, Medieval and Art & Culture Synopsis
1. Ans: D 3. Ans: D
Exp: Exp:
The term mansabdar refers to an individual Aryabhata, the author of the Aryabhatiyam, was
who holds a mansab, meaning a position or associated with the subjects of astronomy,
rank. astronomy, and mathematics:
It was a grading system used by the Mughals Option 1 is correct: Astrology: Though
to fix: Aryabhatt’s primary focus was not on this
Option 1 is correct: Rank was determined by subject, he did make some contributions to
a numerical value called zat; the higher the astrology.
value, the higher the rank. Option 2 is correct: Astronomy: Aryabhata
Option 2 is correct: A numerical value called is renowned for his significant contributions to
zat also fixed the salary of Mansabdars, who the field of astronomy. In his Aryabhatiyam,
were usually paid in jagirs, which yielded he discussed various aspects of astronomy,
revenue equal to the predetermined salary. including planetary motion, eclipses, and the
Option 3 is correct: Military responsibility calculation of celestial phenomena.
was decided by a numerical value called sawar, Option 3 is correct: Mathematics: Aryabhata
which determined the number of horses that a was a brilliant mathematician, and his
mansabdar had to maintain. Aryabhatiyam is a testament to his
Reference: NCERT, Class 7th, Our Pasts II, mathematical prowess. He made
Chapter 4 groundbreaking contributions to algebra,
trigonometry, and arithmetic. His work laid
2. Ans: B the foundation for future developments in
Exp: mathematics.
Statement 1 is incorrect: A Persian chronicler Reference: NCERT, Class 6th, Our Pasts I,
named Minhaj-i-Siraj used the term Hindustan Chapter 10
for the regions of Punjab, Haryana, and the
provinces between the Ganges and the Yamuna. 4. Ans: A
He used the term Hindustan in a political sense Exp:
for lands that were a part of the dominions Pair 1 is correct: Gajapati literally means
of the Delhi Sultan. The areas included in this lord of elephants. This was the name of a
term shifted with the extent of the Delhi ruling lineage that was very powerful in
Sultanate, but the term, as used/connoted by Orissa in the fifteenth century. The Gajapati
him, never included South India. dynasty, under leaders like Kapilendra Deva,
Statement 2 is correct: In the early sixteenth played a significant role in the history of Odisha
century, Babur used Hindustan to describe during that period.
the geography, the fauna, and the culture of Pair 2 is incorrect: In the popular traditions of
the inhabitants of the subcontinent. Vijayanagara, the Deccan Sultans are termed
While the idea of a geographical and cultural as ashvapati, or lords of horses. This term
entity like “India” did exist, the term likely symbolised the military might and
“Hindustan” did not carry the political and equestrian skills of the Deccan Sultans, who
national meanings that we associate with it were often engaged in conflicts and wars with
today. the Vijayanagara Empire.
Reference: NCERT, Class 7th, Our Pasts II, Pair 3 is incorrect: In the context of the
Chapter 1 Vijayanagara Empire, the rulers, who were
known as the Rayas, were sometimes
referred to as "narapati," which means
"lord of men" in Sanskrit. This title reflects
their role as the sovereigns or kings of the

https://upscpdf.com/ https://upscpdf.com/
https://upscpdf.com/ https://upscpdf.com/

empire, symbolising their authority and 7. Ans: A


leadership over their subjects. Exp:
Reference: NCERT, Class 12th, Themes in Statement 1 is incorrect: Nizam-ul-Mulk
Indian History, Part II, Theme 7 Asaf Jah, the founder of Hyderabad state,
was one of the most powerful members at the
5. Ans: C court of Mughal Emperor Farrukh Siyar. He
Exp: was entrusted first with the governorship of
During the Sangam age, Aryabhata, a Awadh, and later given charge of the Deccan.
mathematician and astronomer, wrote a book in Statement 2 is correct: The state of Hyderabad
Sanskrit known as the Aryabhatiyam. was constantly engaged in a struggle against the
He stated that day and night were caused by the Marathas to the west and with independent
rotation of the earth on its axis, even though it Telugu warrior chiefs (nayakas) of the plateau.
seems as if the sun is rising and setting every The Nizam resisted the Maratha claims for
day. Hence, Option 1 is correct. Chauth and Sardeshmukhi in the Deccan and
He developed a scientific explanation for was constantly at war with them.
eclipses as well. Hence, Option 2 is correct. Reference: NCERT,Class 7th, Our Pasts II,
He also found a way of calculating the Chapter 8, Chapter 10
circumference of a circle, that is nearly as
accurate as the formula we use today. Hence, 8. Ans: A
Option 3 is correct. Exp:
Susruta, in his book Susruta Samhita, speaks As warfare during the Vijayanagara empire
about elaborate surgical procedures. Hence, depended upon effective cavalry, the import
Option 4 is incorrect. It is not related to of horses from Arabia and Central Asia was
Aryabhatiyam. very important for rival kingdoms. The
Reference: NCERT, Class 6th, Our Pasts I, Vijayanagara Empire had established trade
Chapter 10 links with regions like Arabia and Central Asia
to procure high-quality horses. These trade
6. Ans: D routes facilitated the exchange of not only
Exp: horses but also other valuable goods and
Statement 1 is incorrect: Mansabdars received commodities. This trade was initially
their salaries as revenue assignments called controlled by Arab traders. Local communities
jagirs, which were somewhat like iqtas. But of merchants known as kudirai chettis, or horse
unlike muqtis, most mansabdars did not merchants, also participated in these exchanges.
actually reside in or administer their jagirs. Thus, the import of horses from Arabia and
Statement 2 is correct: They only had rights Central Asia was a crucial aspect of the
to the revenue of their assignments, which Vijayanagara Empire's military and political
was collected for them by their servants, strategy. Horses were not only valuable for their
while the mansabdars themselves served in military utility but also played a role in trade,
some other part of the country. diplomacy, and culture during this period.
Statement 3 is correct: The mansabdar’s Hence, Both Statement-I and Statement-II
military responsibilities required him to are correct and Statement-II is the correct
maintain a specified number of sawar, or explanation for Statement-I.
cavalrymen. The mansabdar brought his Reference: NCERT, Class 12th, Themes in
cavalrymen for review, got them registered, had Indian History, Part II, Theme 7
their horses branded, and then received money
to pay them as salary. 9. Ans: C
Reference: NCERT, Class 7th, Our Pasts II, Exp:
Chapter 4 Option C is correct:
Silappadikaram-4 (Ilango): Silappadikaram,
a masterpiece composed by the poet Ilango
around 1800 years ago, stands as one of the
earliest Tamil epics. This epic tale revolves
around the life of a merchant named Kovalan.

https://upscpdf.com/ https://upscpdf.com/
https://upscpdf.com/ https://upscpdf.com/

Kovalan's life takes a dramatic turn when he 10. Ans: A


falls in love with the courtesan Madhavi, Exp:
leading to the neglect of his wife, Kannagi. Statement 1 is correct: The term
The story unfolds against the backdrop of "mansabdar" pertains to an individual who
vibrant ancient Tamil Nadu, offering a vivid held a "mansab," signifying a specific
portrayal of their society, culture, and values. position or rank within the Mughal
Manimekalai - 2 (Sattanar): The Tamil epic administrative and military framework. The
Manimekalai, authored by the poet Sattanar Mughals employed a structured grading system
around 1400 years ago, continues the narrative to establish (1) the rank, (2) the salary, and (3)
of Silappadikaram. It introduces us to the the military responsibilities of these officials.
daughter of Kovalan and Madhavi, named Both rank and salary were assigned
Manimekalai. This epic follows her journey as numerical values referred to as "zat." This
she navigates the challenges of life, spirituality, system played a pivotal role in determining the
and destiny. Manimekalai's story is not only a status, roles, and remunerations of individuals
continuation of her parents' tale but also a in the Mughal administration and military.
reflection of the societal changes during that Statement 2 is incorrect: The greater the
period, making this epic an essential part of assigned "zat," the more esteemed the
Tamil literature. noble's position was within the royal court,
Meghaduta - 3 (Kalidasa): It is a celebrated and correspondingly, the larger their salary.
work by the renowned poet Kalidasa, is a Nobles with a zat of 5,000 were ranked higher
Sanskrit poem that narrates the poignant story than those with a zat of 1,000. In Akbar’s
of a monsoon cloud serving as a messenger reign, there were 29 mansabdars with a rank of
between separated lovers. This lyrical 5,000 zat; by Aurangzeb’s reign, the number of
masterpiece, composed approximately 2000 mansabdars had increased to 79.
years ago, showcases Kalidasa's exceptional Reference: NCERT, Class 7th, Our Pasts II,
poetic prowess. In Meghaduta, a Yaksha, a Chapter 4
celestial being, is exiled from his beloved and
must send a message to her through a cloud. 11. Ans: B
Kalidasa's vivid descriptions of natural beauty, Exp:
landscapes, and emotions bring the narrative to Statement 1 is correct: Kushan kings
life. introduced gold and copper coins. A large
Ramayana - 1 (Valmiki):, The revered sage number of these coins have been found by
and poet Valmiki, is credited with the historians. Kushana coins have the figure,
creation of the epic Sanskrit poem name, and title of the king on one side, and on
Ramayana. Composed over 2000 years ago, the reverse side are deities belonging to the
Ramayana narrates the epic journey of Prince Brahmanical, Buddhist, Greek, Roman, and
Rama of Kosala, his exile, and his quest to other pantheons. The legends are either entirely
rescue his kidnapped wife, Sita, from the in Greek or, in some cases, in Kharoshthi, on
demon king Ravana. This monumental work the reverse.
delves into themes of duty, righteousness, and Statement 2 is correct: The reverse of the
the eternal battle between good and evil. Gupta coins has religious symbols indicating
Valmiki's storytelling prowess and moral the kings religious affiliations. There was a
teachings continue to influence not only decline in the metallic purity of gold coins in
Indian literature but also the cultural and the later part of Skandagupta’s reign. The
philosophical heritage of the Indian Guptas also issued silver coins, but their copper
subcontinent coins are rare.
Reference: NCERT, Class 6th, Our Pasts I, Statement 3 is incorrect: The Satavahanas
Chapter 10 supplanted the Mauryas in the Deccan region.
They were first dynasty of Indian origin to issue
coins with the image of the rulers . Punch
marked coins, however, continued in
circulation even in the Satavahana period.

https://upscpdf.com/ https://upscpdf.com/
https://upscpdf.com/ https://upscpdf.com/

Reference: NCERT, Class 12th, Themes in 14. Ans: A


Indian History, Part I, Theme 2 Exp:
Pair 1 is incorrectly matched:
12. Ans: D Silappadikaram, was composed by a poet
Exp: named Ilango Adigal, around 1800 years ago. It
Option D is correct: The Stupa (a Sanskrit is the story of a merchant named Kovalan, who
word meaning a heap) originated as a simple lived in Puhar and fell in love with a courtesan
semi-circular mound of earth, later called an named Madhavi, neglecting his wife Kannagi.
anda. Gradually, it evolved into a more Pair 2 is incorrectly matched: The
complex structure, balancing round and square Manimekalai was composed by Sattanar around
shapes. Above the anda was the harmika, a 1400 years ago. This describes the story of the
balcony-like structure that represented the daughter of Kovalan and Madhavi.
abode of the gods. Arising from the harmika Pair 3 is correctly matched: Kalidasa has
was a mast called the yashti, often surmounted written the book named Meghaduta, in which a
by a chhatri or umbrella. Around the mound monsoon cloud is imagined to be a messenger
was a railing, separating the sacred space from between lovers who are separated from one
the secular world. another.
Reference: NCERT, Class 6th, Our Pasts I,
Chapter 10

15. Ans: C
Exp:
Option C is correct: During Akbar's reign, his
revenue minister, Todar Mal, conducted a
comprehensive survey spanning a decade
(1570-1580), which meticulously documented
crop yields, prices, and cultivated areas.
Based on this extensive data, the tax to be paid
Reference: NCERT, Class 12th, Themes in in cash was fixed for each crop. Each province
Indian History, Part 1, Theme 4 was subdivided into revenue circles, each with
its own schedule of revenue rates for various
13. Ans: C crops. This systematic approach to revenue
Exp: collection was termed "zabt." It was primarily
Statement 1 is correct: The "Lilatilakam" is implemented in regions where Mughal
a prominent medieval treatise on grammar administrators could effectively survey the land
and poetry in India. and maintain precise records. However, it was
Statement 2 is correct: The Lilatilakam was not feasible to apply this system in provinces
written in Manipravalam by a poet and like Gujarat and Bengal.
scholar named Thunchaththu Ramanujan Reference: NCERT, Class 7th, Our Pasts II,
Ezhuthachan. He is often referred to as the Chapter 4
"Father of Malayalam Language and
Literature." He composed this work in the 14th 16. Ans: D
century CE in the region of Kerala, India. Exp:
Lilatilakam was composed in Manipravalam, Statement 1 is correct: The Kharosthi script
literally, “diamonds and corals,” referring to the (also known as 'Indo-Bactrian' script) was a
two languages, Sanskrit and the regional writing system originally developed in
language. present-day northern Pakistan, sometime
Reference: NCERT, Class 7th, Our Pasts II, between the 4th and 3rd centuries BCE. It was
Chapter 7 employed to represent a form of Prakrit, an
Indo-Aryan language. It was written from
right to left.

https://upscpdf.com/ https://upscpdf.com/
https://upscpdf.com/ https://upscpdf.com/

Statement 2 is correct: The Brahmi script is 19. Ans: B


the earliest writing system developed in India Exp:
after the Indus script. Most scripts used to write Statement 1 is correct: Akbar’s revenue
modern Indian Languages are derived from minister, Todar Mal, carried out a careful
Brahmi, the script used in most Ashokan survey of crop yields, prices, and areas
Inscriptions. It was written from left to right, cultivated for a 10-year period, 1570- 1580. On
and James Prinsep deciphered the Ashokan the basis of this data, taxes were fixed on each
Brahmi in 1838. crop in cash.
Statement 3 is correct: Brahmi scripts Statement 2 is incorrect:Zabt system was
prevailed in the whole country except for the prevalent in those areas where Mughal
north-western part. Greek and Aramaic scripts administrators could survey the land and keep
were employed in writing Ashokan Inscriptions very careful accounts. This was not possible in
in Pakistan and Afghanistan. Brahmi provinces such as Gujarat and Bengal.
continued to be the main script until the end Statement 3 is correct: Each province was
of the Gupta period. divided into revenue circles with its own
Reference: NCERT, Class 12th, Themes in schedule of revenue rates for individual
Indian History, Part I, Theme 2 crops. This revenue system was known as
Zabt.
17. Ans: A Reference: NCERT, Class 7th, Our Pasts II,
Exp: Chapter 4
Statement 1 is incorrect: In the early medieval
times, as society became more differentiated, 20. Ans: B
people were grouped into jatis, or sub-castes Exp:
and ranked on the basis of their backgrounds Option B is correct: The cult of Jagannatha
and their occupations. Jatis framed their own in Puri, Odisha, India, is primarily devoted
rules and regulations to manage the conduct of to Lord Jagannatha, who is considered to be
their members. an incarnation of Lord Vishnu
Statement 2 is correct: The rules and (Krishna). To date, the local tribal people
regulations that were framed by the jatis were make the wooden image of the deity, which
enforced by an assembly of elders, described in suggests that the deity was originally a local
some areas as the jati panchayat. god, who was later identified with Vishnu.
Statement 3 is incorrect: Jatis were also Reference: NCERT, Class 7th, Our Pasts II,
required to follow the rules of their villages, as Chapter 7
some of them were governed by chieftains.
Reference: NCERT, Class 7th, Our Pasts II, 21. Ans: C
Chapter 1 Exp:
Option C is correct: The Sudarshana lake, a
18. Ans: B man-made reservoir, is illuminated through a
Exp: Sanskrit Girnar inscription (circa second
Option B is correct: After Chandragupta, his century CE) chronicling Shaka ruler
son Samudragupta (A.D. 335–375) ascended Rudradaman's accomplishments. It tells us
the throne and pursued a policy of conquest, that the lake with walls and water paths was
significantly expanding the kingdom. initially built during the time of the Maurya
Samudragupta, in addition to being a rulers. Unfortunately, a big storm wrecked the
conqueror, was a multifaceted individual. He walls, and water rushed out. Rudradaman, who
was a poet, a musician, and a patron of learning. was the ruler back then, took charge and fixed
His passion for music is evident from the coins the lake using his own money, without
bearing his image, where he is portrayed burdening his people with taxes. Another
playing the vina (lute). inscription on the same rock (around the 5th
Reference: NCERT, Class 6th, Our Pasts-I, century) mentions that a Gupta dynasty ruler
Chapter 9 also repaired the lake.
Reference: NCERT, Class 12th, Themes in
Indian History, Part I, Theme 2

https://upscpdf.com/ https://upscpdf.com/
https://upscpdf.com/ https://upscpdf.com/

22. Ans: A from the Chalukyas by overthrowing their last


Exp: ruler, Kirtivarman II.
Option A is correct: Mehrunnisa married the Statement 3 is correct: Dantidurga was a
Emperor Jahangir in 1611 and received the title Rashtrakuta ruler who overthrew his
Nur Jahan. She remained extremely loyal and Chalukya overlord and performed a ritual
supportive of the monarch. As a mark of called hiranya-garbha (literally, the golden
honour, Jahangir struck silver coins bearing womb), establishing the Rashtrakuta dynasty as
his own titles on one side, and on the other side an independent power. Under his leadership,
was the inscription, “struck in the name of the the Rashtrakutas gained autonomy and
Queen Begum, Nur Jahan”. eventually dominated the Deccan region.
Reference: NCERT, Class 7th, Our Pasts II, Reference : NCERT, Class 7th, Our Pasts II,
Chapter 4 Chapter 2

23. Ans: A 25. Ans: C


Exp: Exp:
Option A is correct: Across the expanse of Option C is correct: During the Gupta era, the
time, India's profound quest for understanding kings took various measures to gain the support
reality has crystallised into six distinct orthodox of influential individuals, whether due to their
schools of philosophy. These are: economic, social, political, or military power.
Schools of Philosophy Founded By Moreover, these influential figures likely
Vaisheshika Konada played a role in local administration. These
Nyaya Gotama included the chief banker or merchant of the
Samkhya Kapila city (nagarashreshthi), the leader of merchant
Yoga Patanjali caravans (sarthavaha), the chief craftsman
Mimansa Jaimini (prathama-kulika), and the head of
Vedanta Vyasa scribes/kayasthas.
These philosophies still guide scholarly Reference: NCERT, Class 6th, Our Pasts I,
discourse in the country. German-born Chapter 9
British indologist, Friedrich Max Muller, has
observed that the six systems of philosophy 26. Ans: A
were developed over many generations with Exp:
contributions made by many individual Arthashastra, written by Kautilya, describes
thinkers. how spinning and weaving could be done in
Reference: NCERT, Class 6th, Our Pasts-I, workshops under the supervision of a special
Chapter 6 official. “Widows, young women who are
differently abled, nuns, mothers of courtesans,
24. Ans: B retired women servants of the king, and women
Exp: who have retired from service in temples, may
Statement 1 is incorrect: The Rashtrakutas be used for processing wool, bark, cotton,
were a dynasty that ruled over Deccan parts hemp, and flax. Rules are:
(not the Malwa region) of the Indian • They should be paid according to the
subcontinent from the 6th to the 10th centuries quality and quantity of their work.
CE. They were known for their significant Hence, Statement 1 is correct.
contributions to art, culture, and architecture, • Women who are not permitted to
particularly in the Deccan region of India. leave their homes can send
Statement 2 is correct: The Rashtrakuta maidservants to bring the raw materials
dynasty was initially a subordinate dynasty from the superintendent and take the
to the Chalukyas of Badami, who were the finished work back to him. Hence,
dominant power in the Deccan region during Statement 2 is incorrect.
that time. The Rashtrakutas gained prominence Reference: NCERT, Class 6th,Our Pasts I,
under their early rulers, and then it was Chapter 8
Dantidurga who managed to gain independence

https://upscpdf.com/ https://upscpdf.com/
https://upscpdf.com/ https://upscpdf.com/

27. Ans: C 29. Ans: A


Exp: Exp:
The central government in the Mughal Statement 1 is correct: A Bodhisattva is
administration was headed by the Mughal indeed a compassionate person who is on the
emperor, and the ministers worked immediately path to enlightenment. In Buddhism,
beneath him. particularly Mahayana Buddhism, a
Pair 1 correctly matched: The entire nobility Bodhisattva is an individual who aspires to
or ruling elite were called Umara during the attain Buddhahood, the highest state of
Mughal empire. They belong to the highest rank enlightenment, but delays their own
of the Mansabdari system. liberation from the cycle of suffering
Pair 2 incorrectly matched: Under the Mughal (samsara) to help all sentient beings achieve
Empire, Faujdar was a rank referring to liberation as well. This aspiration is grounded
Military officers. They have the responsibility in boundless compassion and altruism.
of protecting the territory. Statement 2 is correct: This statement is also
Pair 3 correctly matched : Subadar is the accurate. A central characteristic of a
rank equivalent to governor, which originally Bodhisattva is their willingness to postpone
referred to the senior officer of the Mughal their own salvation or enlightenment to assist
Empire. The Mughal empire was divided into a all living beings in reaching it. They commit to
number of subahs (provinces). Each of the continuously reincarnating in various forms
subas was headed by a provincial governor. He until all sentient beings have been liberated
was called a subahdar. from suffering and have attained
Pair 4 correctly matched : Chauth basically enlightenment. This selfless dedication to the
referred to 25 percent of the land revenue well-being of others is a fundamental aspect of
claimed by zamindars. In the Deccan, this was the Bodhisattva ideal.
collected by the Marathas. Chauth is a word Statement 3 is incorrect: The concept of a
from the Sanskrit language, and it means one bodhisattva is not an important aspect of
fourth. Hinayana Buddhism. It is primarily
Reference: NCERT, Class 7th, Our Pasts II, associated with Mahayana Buddhism. In
Chapter 8 Hinayana Buddhism, also known as
Theravada Buddhism, the emphasis is more
28. Ans: A on individual liberation and the path to
Exp: becoming an arhat, a perfected or enlightened
Terms Used for different social classes in south being. While compassion is valued in
India second century BC onwards: Theravada Buddhism, the Bodhisattva ideal,
• Arasar- Ruling Class with its commitment to the liberation of all
• Vellalar- Rich Peasants. Hence, 1 is beings, is a distinctive feature of Mahayana
incorrectly matched Buddhism. Therefore, In Hinayana Buddhism,
• Kadaisiyars and Adimai- slaves and the focus is more on personal liberation from
landless labourers Hence, 3 is suffering, and the role of the Bodhisattva is not
correctly matched as prominent as it is in Mahayana Buddhism.
• Uzhavar- Ordinary Ploughmen Reference: NCERT, Class 12th, Themes in
Hence, 2 is correctly matched Indian History I, Theme 4
• Pulaiyans- Artisans who made rope
Charpai 30. Ans: A
• Enadi- Captains of army Exp:
Sharp social inequalities are evident in the age Statement-I is correct: The Jagannath Temple
of Sangam, rich people lived in houses of brick in Puri has long been a symbol of cultural and
and mortar and poor people lived in humbler religious identity for the people of Odisha. As
structures, but it is not known clearly whether the temple gained in importance as a centre of
religion was the sanction behind social pilgrimage, its authority in social and political
inequalities. matters also increased. All those who
Reference: NCERT, Class 6th, Our Pasts I, conquered Orissa, such as the Mughals, the
Chapter 8.

https://upscpdf.com/ https://upscpdf.com/
https://upscpdf.com/ https://upscpdf.com/

Marathas, and the English East India Company, - Sites such as Dholavira and Lothal
attempted to gain control over the temple. (Gujarat), the entire settlement was fortified,
Statement-II is correct: Mughals, the and sections within the town were also
Marathas, and the English East India Company separated by walls.
felt that this would make their rule acceptable - The Citadel within Lothal was not walled
to the local people. off, but was built at a height.
The Mughal Empire, which ruled over much of Statement 2 is incorrect: Important features of
the Indian subcontinent in the medieval and the Great Bath discovered at Mohenjo-Daro:
early modern periods, showed an interest in the • There was a large rectangular tank in
Jagannath Temple as part of their efforts to the courtyard, surrounded on all four
consolidate control over different regions. sides by Corridors.
The Marathas, a regional power in western • Two flights of steps in the north and
India, extended their influence to various parts south, leading into the tank.
of India, including Odisha. They, too, • The tank was made watertight by
recognised the religious and cultural setting bricks on edge,
significance of the Jagannath Temple and - Rooms on three sides, one of which had a well.
sought to establish their authority by gaining -Smaller building towards the north, with eight
control over it. Bathrooms, four on either side of the corridor.
As the English East India Company expanded -Scholars suggest that it might have been used
its influence in India, including the eastern for some kind of ritual bath.
regions, they encountered the Jagannath Statement 3 is correct: In the Indus Valley
Temple and recognised its importance in the Civilization, Harappa was the first site to be
local culture. discovered, in 1921, by Dayaram Sahni, an
Hence, Statement-II is the correct Indian archaeologist.
explanation for Statement-I Rai Bahadur Dayaram Sahni was also the first
Reference: NCERT, Class 7th, Our Pasts II, archaeologist to be appointed Director-
Chapter 7 General of the Archaeological Survey Of
India.
31. Ans: C Reference: NCERT,Class 12th, Themes in
Exp: Indian History, Part I, Chapter 1.
Option C is correct: Between the sixth and
fourth centuries BCE, Magadha (in present- 33. Ans: B
day Bihar) became the most powerful Exp:
mahajanapada. Initially, Rajagaha (the Nadir Shah, the Shah of Persia (Iran),
Prakrit name for present day Rajgir in invaded India in 1739, defeated Muhammad
Bihar) was the capital of Magadha. Shah, the Mughal Emperor, and took away the
Interestingly, the old name means “house of the famous Kohinoor diamond, the Peacock
king”. Rajagaha was a fortified settlement Throne of Shahjahan, and a huge wealth. With
located among the hills. However, in the fourth the looted wealth, he was able to exempt taxes
century BCE, the capital was relocated to in his kingdom for three years. Nizam-ul-Mulk
Pataliputra, contemporary Patna, a strategic and Saadat Khan were rulers of the states of
move that facilitated control over Hyderabad and Awadh at that time. They
communication routes along the Ganga fought on the side of the Mughals in the Battle
river. of Karnal, fought between the Mughals and
Reference: NCERT, Class 12th, Themes in Nadir Shah in 1739.
Indian History, Part I, Theme 2 The Koh-i-Noor (Mountain of Light), is one of
the largest cut diamonds in the world. It weighs
32. Ans: C 21.12 grams, and It is currently a part of the
Exp: Crown Jewels of the United Kingdom.
Statement 1 is correct: Most of the Harappan Reference: NCERT, Class 7th, Our Pasts II,
settlements had a small high western part and a Chapter 8
larger lower eastern section, but there were
variations:

https://upscpdf.com/ https://upscpdf.com/
https://upscpdf.com/ https://upscpdf.com/

34. Ans: C 37. Ans: D


Exp: Exp:
Options Aramaic, Kharoshti, and Brahmi Under a number of able leaders in the
are correct: eighteenth century, Sikhs instituted a system of
Most Ashokan inscriptions were in the Prakrit administration.
language, while those in the northwest of the Pair 1 correctly matched: The Sikhs
subcontinent were in Aramaic and Greek organised themselves into a number of bands
language. Most Prakrit inscriptions were called jathas, and later on misls.
written in the Brahmi script; however, some, in Pair 2 correctly matched: Their combined
the northwest, were written in Kharosthi forces were known as the grand army (dal
Script. The Aramaic and Greek scripts were khalsa).
used for inscriptions in Afghanistan. Pair 3 correctly matched: The entire body
Reference: NCERT, Class 12th, Themes in used to meet at Amritsar at the time of Baisakhi
Indian History, Part I, Theme 2 and Diwali to take collective decisions known
as “resolutions of the Guru (gurmatas)”.
35. Ans: B Pair 4 correctly matched: A system called
Exp: rakhi was introduced, offering protection to
Chola Inscriptions from Tamil Nadu refer to cultivators on payment of a tax of 20 percent of
more than 400 terms for various kinds of taxes; the produce.
Vetti among them stood out as it was not a tax Reference: NCERT, Class 7th, Our Pasts II,
that was collected in kind, rather it was a tax Chapter 8
appropriated in the form of forced labour.
Other kinds of taxes that were collected were 38. Ans: B
Kadamai (Land Revenue), taxes on thatching Exp:
the houses,, taxes on tapping toddy, etc. Option B is correct: We know in detail about
Reference : NCERT, Class 7th, Our Pasts II, Gupta King Samudragupta from a long
Chapter 2 inscription, engraved upon An Ashokan Pillar
in Allahabad (also known as Prayaga
36. Ans: A Prashasti). The inscription is actually a poem
Exp: in Sanskrit, composed by his court poet,
Mathura has been an important settlement for Harishena nearly 1700 years ago.
more than 2500 years. It was important because Reference: NCERT, Class 6th, Our Pasts I,
it was located at the crossroads of two major Chapter 9
routes of travel and trade — from the
northwest to the east and from the north to 39. Ans: C
the south. There were fortifications around the Exp:
city and several shrines. Farmers and herders Option C is correct: The Ganga dynasty was a
from adjoining areas provided food for people medieval Indian dynasty that ruled parts of
in the city. Mathura was also a centre where present-day Odisha, and they played a
some extremely fine sculpture was produced. significant role in the temple's architectural
Around 2000 years ago, Mathura became the development, maintenance, and religious
second capital of the Kushanas. Mathura was practices.
also a religious centre; there were Buddhist In the twelfth century, one of the most
monasteries, Jaina shrines, and it is now an important rulers of the Ganga dynasty,
important centre for the worship of Krishna. Anantavarman, decided to erect a temple for
Reference: NCERT, Class 6th, Our Pasts I, Purushottama Jagannatha at Puri.
Chapter 8 Subsequently, in 1230, King Anangabhima III
dedicated his kingdom to the deity and
proclaimed himself the “deputy” of the god.
Reference: NCERT, Class 7th, Our Pasts II,
Chapter 7

https://upscpdf.com/ https://upscpdf.com/
https://upscpdf.com/ https://upscpdf.com/

10

40. Ans: B Sawai Jai Singh, the ruler of Amber,


Exp: constructed five astronomical observatories,
Statement 1 is incorrect: Indo-Greeks or the one each in Delhi, Jaipur, Ujjain, Mathura,
Bactrian Greeks, were the first to issue gold and Varanasi. Commonly known as Jantar
coins in India, which increased in number later Mantar, these observatories were used for
under Kushanas. making astronomical observations.
Statement 2 is correct: The first coins that can Reference: NCERT, Class 7th, Our Pasts II,
be attributed to kings were issued by the Chapter 8
Indo-Greeks/Bactrian Greeks, who
established control over the north-western part 43. Ans: A
of the subcontinent in the second century BCE. Exp :
Punch marked coins issued earlier could not be Statement 1 is correct: The Iranians and the
attributed to kings and dynasties. Greeks who came through the Northwest
Statement 3 is incorrect: A number of tribal about 2500 years ago were familiar with the
republics existed in Punjab. They had lost their river Indus; they called it ‘the Hindos’ or ‘the
independence during the Maghdan Empire, but Indos’, and the land to the east of the river was
as soon as the empire collapsed, they returned called India. The term 'India' has been derived
to their own republican system and started from it.
issuing coins. The tribes that issued their coins Statement 2 is incorrect: The name Bharata
in the second century BCE were the Agreya, was used for a group of people who lived in
Kshudraka, Rajanya, Sibi, Trigarta, and the Northwest of the subcontinent and who
Yaudheya. are mentioned in the Rig Veda, the earliest
Reference: NCERT, Class 6th, Our Pasts I, composition in Sanskrit (dated to about 3500
Chapter 4. years ago). Later, the term was used to denote
the country. So the word 'Bharata' is found in
41. Ans: B the historical texts prior to the Mahabharata
Exp: period.
Option B is correct The Sutta Pitaka is one of Reference: NCERT, Class 6th, Our Pasts I,
the three main sections, or "baskets," of the Chapter 1
Pali Canon, which is the foundational text of
Theravada Buddhism. It consists of a vast 44. Ans: C
collection of discourses attributed to the Exp:
Buddha, his teachings, and interactions with The wall relief showing Vishnu as Narasimha
various individuals, including rulers like King is a notable feature found in the Ellora Caves,
Ajatasattu. which are a series of rock-cut temples and
The specific conversation between King monasteries located in the Indian state of
Ajatasattu and the Buddha is often found in Maharashtra. The relief is a work from the
the "Digha Nikaya" (The Long Rashtrakuta period, between the 6th and 10th
Discourses) of the Sutta Pitaka. These texts centuries CE.
contain dialogues and discourses that provide
deep insights into Buddhist teachings and
philosophy.
Reference: NCERT, Class 12th, Themes in
Indian History I, Theme 4

42. Ans: C
Exp:
Jai Singh II was popularly known as Sawai Raja
Jai Singh. He was a Kachwaha ruler of the
Kingdom of Amber, and he founded the
fortified city of Jaipur and made it his capital (Fig. The wall relief shows Vishnu as Narasimha)
city. Reference: NCERT, Class 7th , Our Pasts II,
Chapter 2

https://upscpdf.com/ https://upscpdf.com/
https://upscpdf.com/ https://upscpdf.com/

11

45. Ans: D meant to be accessible to a wider audience,


Exp: including common people who may not have
Options Taxila, Ujjayini, Tosali, Suvarnagiri been scholars or priests. Additionally, the
are all correct: There were five major political Puranas were often recited or read aloud in
centres in the empire – the capital Pataliputra public gatherings or during religious
and the provincial centres/capitals of Taxila, ceremonies, making them more engaging and
Ujjayini, Tosali and Suvarnagiri, all understandable as stories of deities and their
mentioned in Asokan inscriptions. exploits.
These centres were carefully chosen, both Statement 2 is incorrect : Vaishnavism does
Taxila and Ujjayini being situated on important not proclaim that there are one hundred avatars
long-distance trade routes, while Suvarnagiri of Vishnu. Instead, Vaishnavism primarily
(literally, the golden mountain) was possibly recognizes ten principal avatars of Lord
important for tapping the gold mines of Vishnu, known as the Dashavatara. These ten
Karnataka. avatars include prominent incarnations like
Tosali is believed to have been located in the Lord Rama and Lord Krishna. The concept
modern state of Odisha, India. During the of one hundred avatars is not a standard belief
Ashokan period, it was one of the regional within the Vaishnavism tradition.
centres in the Mauryan Empire. Statement 3 is correct : Shaivism, one of the
Reference: NCERT 12th Class: NCERT, major sects within Hinduism, does indeed use
Class 12th, Themes in Indian History, Part I, various symbols to represent Lord Shiva. One
Theme 2 of the most prominent symbols is the Linga
(or Shiva Linga), which is a symbol
46. Ans: A representing the divine energy and the
Exp: formless aspect of Shiva. Additionally, Shiva
Statement 1 is incorrect: Jats were is also represented in human form as a deity
predominantly agriculturalists in north in various temples and iconography. Therefore,
India. They consolidated their power in the Shaivism does symbolize Shiva both as a
late seventeenth and early eighteenth centuries human figure and as the Linga.
under their leader, Churaman. They gained Reference: NCERT, Class 12th, Themes in
control of territories to the west of Delhi and Indian History I, Theme 4
dominated the region between Delhi and Agra.
For a while, they became the virtual custodians 48. Ans: B
of the city of Agra. They also established the Exp:
Bharatpur State. Option B is correct: Therigatha unique
Statement 2 is correct: Suraj Mal was one of Buddhist text, a part of the Sutta Pitaka, one of
the prominent Jat leaders who united the the earliest books of the Pāli canon, is a
several Jat tribes into a single great empire. collection of verses also known as the Hymns
Under Suraj Mal, the kingdom of Bharatpur (a of the Elders/Senior Nuns known as
Jat state) emerged as a strong state. bhikkhunis. It provides an insight into
Reference: NCERT, Class 7th, Our Pasts II, women’s social and spiritual experiences.
Chapter 8 The word ‘Theri’ was used for respected
woman who had obtained liberation.
47. Ans: B Initially, only men were allowed to join the
Exp: Sangha. Women were allowed to join Sangha
Statement 1 is correct : The Puranas are a later as Bhikkunis only after mediation by
genre of ancient Indian texts that contain a wide Ananda on their behalf. Mahapajapati Gotami,
range of stories, myths, legends, and religious Lord Buddha’s foster mother, was first woman
teachings. They were written in Sanskrit, but to be ordained ‘Bhikkuni”.
the language used in Puranas is relatively Reference: NCERT, Class 12th, Themes in
simpler and more accessible compared to the Indian History, Part I, Theme- 4
classical Sanskrit used in other texts like the
Vedas and Upanishads. This simplification of
language was intentional, as the Puranas were

https://upscpdf.com/ https://upscpdf.com/
https://upscpdf.com/ https://upscpdf.com/

12

49. Ans: D earliest to produce cotton, hence, the Greeks


Exp: called cotton, Sindon (derived from Sindh).
Tribal people were found in almost every Reference: NCERT, Class 12th, Themes in
region of the subcontinent. The area and Indian History, Part I, Chapter 1.
influence of a tribe varied at different points in
time. Some powerful tribes controlled large 51. Ans: B
territories. Exp:
Pair 1 is correct: In Punjab, the Khokhar tribe Pair 1 is incorrectly matched: Verses ascribed
was very influential during the thirteenth and to Kabir have been compiled in three distinct
fourteenth centuries. Later, the Gakkhars but overlapping traditions. The Kabir Bijak is
became more important. Their chief, Kamal preserved by the Kabirpanth (the path or sect of
Khan Gakkhar, was made a noble (mansabdar) Kabir) in Varanasi and elsewhere in Uttar
by Emperor Akbar. Pradesh; the Kabir Granthavali is associated
Pair 2 is correct: The distant north-eastern part with the Dadupanth in Rajasthan, and many of
of the subcontinent too was entirely dominated his compositions are found in the Adi Granth
by tribes – the Nagas, Ahoms, and many others. Sahib. All these manuscript compilations were
Pair 3 is correct: In Multan and Sind, the made long after the death of Kabir.
Langahs and Arghuns dominated extensive Pair 2 is correctly matched: Jayadeva of
regions before they were subdued by the Bengal composed the Gita Govinda in
Mughals. Sanskrit, each song composed in a particular
Reference: NCERT, Class 7th, Our Pasts II, raga and tala. He was one of those saints who
Chapter 5 had a significant impact on music through the
use of bhajan, kirtan, and abhang. These songs,
50. Ans: C which emphasised emotional experience, had
Exp: tremendous appeal to the common people.
Agriculture was prevalent in the Harappan Pair 3 is incorrectly matched: Surdas was an
civilization, as supported by the findings ardent devotee of Krishna. His compositions,
of wide prevalence of grains at various sites: compiled in the Sursagara, Surasaravali, and
Statement 1 is correct: Archaeologists have Sahitya Lahari, express his devotion.
found evidence of a ploughed field at Pair 4 is correctly matched:: Padmavat,
Kalibangan (Rajasthan), associated with Early composed by Malik Muhammad Jayasi,
Harappan levels, where the field had two sets of revolved around the romance of Padmini and
furrows at right angles to each other, suggesting Ratansen, the king of Chittor. Their trials were
that two different crops were grown together symbolic of the soul’s journey to the divine.
(intercropping). Such poetic compositions were often recited in
Statement 2 is correct: Representations on hospices, usually during sama‘.
seals and terracotta sculptures indicate that the Reference: NCERT, Class 12th, Themes in
bull was known, and archaeologists have Indian History, Part II, Theme 6
extrapolated from this that oxen were used for
ploughing. Moreover, terracotta models of the 52. Ans: D
plough have been found at sites in Cholistan Exp:
and Banawali (Haryana). Statement 1 is correct: Prabhavati Gupta
Statement 3 is correct: Most Harappan sites was the daughter of one of the most important
are located in semi-arid lands, where irrigation rulers in early Indian history, Chandragupta II
was probably required for agriculture. Traces of (c. 375-415 CE). She was married into
canals have been found at the Harappan site of another important ruling family, that of the
Shortughai in Afghanistan. Vakatakas, who were powerful in the Deccan.
Variety of crops were grown in the Indus Valley Statement 2 is correct: Sanskrit legal texts,
region, including wheat (two types), barley ( for instance, typically did not grant women
two types), rice (evidence at Lothal), pulses, independent access to resources like land.
linseed, sesamum, mustard, and cotton. The However, it's worth noting that exceptions
people of the Indus Valley Civilization were the existed, as seen in the case of Queen Prabhavati,
who had access to and could grant land, which

https://upscpdf.com/ https://upscpdf.com/
https://upscpdf.com/ https://upscpdf.com/

13

was somewhat unusual for her time. The was at Khajuraho, which was constructed by
evidence, both from inscriptions and texts, their feudatory chiefs, the Chandellas, and is
indicates that while upper-class women might now a UNESCO World Heritage Site.
have had some access to resources, resources Pair 2 is incorrect: Rashtrakutas, meaning ‘the
like land, cattle, and money were typically chief of a rashtra’, were considered feudatory of
under male control. the Chalukyas. Dantidurga founded the
Statement 3 is correct: During the Gupta Rashtrakuta kingdom in 753 CE. and they
period, there were regional variations in the ruled over parts of Maharashtra and
sizes of land donated – ranging from small plots Karnataka.
to vast stretches of uncultivated land – and the Pair 3 is incorrect: In 750 AD, Gopala
rights given to donees (the recipients of the founded the Pala dynasty. Palas were followers
grant). An agrahara was land granted to a of Mahayana Buddhism and Tantric Buddhism.
Brahmana, who was usually exempt from They had close cultural ties with Tibet. The
paying land revenue and other dues to the noted Buddhist scholars of their age,
king, and the donee was often given the right Santarakshita and Dipankara, were invited to
to collect these dues from the local people Tibet. They introduced a new form of religion
Reference: NCERT, Class 12th, Themes in there. Pala's kingdom included Bengal and
Indian History, Part I, Theme 2 Bihar, which included the major cities of
NCERT, Class 12th, Themes in Indian History, Pataliputra, Vikrampura, Monghyr, and
Part I, Theme 3 Tamralipti.
Pair 4 is correct: The Chera dynasty ruled
53. Ans: B over the southwestern region of the Indian
Exp: subcontinent, in modern-day Kerala and
Statement 1 is incorrect: Punch-marked coins Tamil Nadu, with its capital at Vanji
were not inscribed, but were stamped with (Malabar). It’s territory is located west and
symbols using dies or punches. Hence, they are north of Pandya. It had a bow and arrow sign as
called punch-marked coins. Punch-marked its emblem. Senguttuvan (Red Chera) was the
coins were generally rectangular or sometimes most powerful ruler. He founded the Kannagi
square or round in shape, either cut out of metal or Pattini Cult, and Kannagi became an object
sheets or made out of flattened metal globules. of worship.
These coins are found in most parts of the
subcontinent and remained in circulation till the
early centuries CE.
Statement 2 is correct: In the villages, there
lived dasa karmakara, who did not own land,
and had to earn a living working on the fields
owned by others. Large landowners were
known as vellalar, and they employed Dasa
Karmakara.
Reference: NCERT, Class 6th, Our Pasts I,
Chapter 9

54. Ans: B
Exp:
Pair 1 is correct: Gurjara – Pratihara, as they
originated from Gurjaras, who were primarily
pastoralists and fighters. The dynasty was
founded by Harichandra, and ruled over the
present states of Madhya Pradesh and
Rajasthan. Gurjara-Pratihara are known for
their sculptures, carved panels, and open
Reference : NCERT, Class 7th, Our Pasts II,
pavilion style temples. The greatest
Chapter 2
development of their style of temple building

https://upscpdf.com/ https://upscpdf.com/
https://upscpdf.com/ https://upscpdf.com/

14

55. Ans: A glorify their spiritual genealogies. Many details


Exp: are often implausible, full of elements of the
Statement 1 is incorrect: Ashoka’s dhamma fantastic. Still, they are of great value to
prescribed non-performance of sacrifices historians and help them understand more fully
and rituals. He felt that just as a father tries to the nature of the tradition.
teach his children, he had a duty to instruct his Reference: NCERT, Class 12th, Themes in
subjects. Indian History, Part II, Theme 6
Statement 2 is correct: During Ashoka's reign,
there were a number of problems that troubled 57. Ans: B
him. People in the empire followed different Exp:
religions, and this sometimes led to conflict. The Rigvedic society was more tribal and
Animals were sacrificed. Slaves and servants decentralised, and the term "rajas" in this
were ill-treated. Ashoka felt it was his duty to context is associated with leadership within
solve these problems. So, he appointed these early tribal communities.
officials, known as the dhamma mahamatta Statement 1 is incorrect: Unlike the organised
who went from place to place teaching cities and capital centres that characterised later
people about dhamma. kingdoms and empires, the Rigvedic period was
Statement 3 is correct: Ashoka also sent marked by a largely pastoral and semi-nomadic
messengers to spread ideas about dhamma society. People lived in small, scattered
to other lands, such as Syria, Egypt, and settlements rather than large cities. The concept
Greece, and his son Mahendra and daughter of grand palaces and fortified capital cities
Sanghmitra to Sri Lanka. He built roads, dug was not prevalent during this time.
wells, and built rest houses. Besides, he Statement 2 is incorrect: The Rigvedic
arranged for medical treatment for both human society did not have standing armies or
beings and animals. professional military forces as we see in later
Reference: NCERT, Class 6th, Our Pasts, historical periods. The people of this time
Chapter 7 were often engaged in agricultural and pastoral
activities. In times of conflict, able-bodied men
56. Ans: D from the community would come together to
Exp: defend against external threats, but this was not
Option D is correct: A wide range of Sufi a centralised and organised army structure.
texts were produced in and around Sufi Statement 3 is correct: The Rigvedic society
khanqahs. These included: did not have a centralised taxation system in
Malfuzats (written conversations of saints) the way that later kingdoms and empires did.
were compiled by different Sufi silsilas with the Economic activities were more localised and
permission of the shaikhs. These had self-sustaining, often centred around
obvious didactic purposes. Several examples agriculture, animal husbandry, and trade within
have been found in different parts of the smaller communities. Rajas/ Chiefs received
subcontinent, including the Deccan. They were offerings in the form of voluntary Bali.
compiled over several centuries. Statement 4 is correct: In the Rigvedic
Maktubat were letters written by Sufi society, leadership positions, including that
masters, addressed to their disciples and of "rajas," were not typically passed down
associates. While these tell us about the through a strict hereditary succession. While
Shaikh’s experience of religious truth that he lineage and family connections did play a role,
wanted to share with others, they also reflect the leadership was often earned based on personal
life conditions of the recipients and are qualities, such as wisdom, valour, and the
responses to their aspirations and difficulties, ability to provide for the community.
both spiritual and mundane. Reference: NCERT, Class 6th, Our Pasts I,
Tazkiras were biographical accounts of Sufi Chapter 4.
saints. The most famous tazkira is the Akhbar-
ul-Akhyar of Abdul Haqq Muhaddis Dehlavi.
The authors of the tazkiras often sought to
establish the precedence of their own orders and

https://upscpdf.com/ https://upscpdf.com/
https://upscpdf.com/ https://upscpdf.com/

15

58. Ans: C image worship, austerities, and the


Exp: scriptures of both Hindus and Muslims. For
Statement 1 is correct: The Gonds lived in a Baba Guru Nanak, the Absolute, or rab,” had no
vast forested region called Gondwana, or gender or form.
“country inhabited by Gonds”. They practised Statement 2 is correct: He proposed a simple
shifting cultivation. way to connect to the Divine by remembering
Statement 2 is correct: The large Gond tribe and repeating the Divine Name, expressing
was further divided into many smaller clans. his ideas through hymns called “shabad ” in
Each clan had its own raja or rai. About the time Punjabi, the language of the region. He
that the power of the Delhi Sultans was organised his followers into a community. He
declining, a few large Gond kingdoms were set up rules for congregational worship
beginning to dominate the smaller Gond chiefs. (sangat) involving collective recitation. Guru
Statement 3 is correct: The Akbar Nama, a Nanak appointed one of his disciples, Angad,
history of Akbar’s reign, mentions the Gond to succeed him as the preceptor (guru), and
kingdom of Garha Katanga, which had 70,000 this practise was followed for nearly 200 years.
villages. Statement 3 is correct: The fifth preceptor,
Reference: NCERT, Class 7th, Our Pasts II, Guru Arjan , compiled Baba Guru Nanak’s
Chapter 5 hymns along with those of his four successors
and other religious poets like Baba Farid,
59. Ans: B Ravidas (also known as Raidas), and Kabir in
Exp: the Adi Granth Sahib. These hymns, called
The structure we are talking about in the “Gurbani”, are composed in various languages.
paragraph is the Great Bath, which is present In the late seventeenth century, the tenth
in Mohenjo-Daro. The Great Bath was a large preceptor,
rectangular tank in a courtyard surrounded by a Guru Gobind Singh, included the
corridor on all four sides. Across a lane to the compositions of the ninth guru, Guru Tegh
north lay a smaller building with eight Bahadur, and this scripture was called the Guru
bathrooms, four on each side of a corridor, Granth Sahib. Guru Gobind Singh also laid the
with drains from each bathroom connecting foundation of the Khalsa Panth (army of the
to a drain that ran along the corridor. pure) and defined its five symbols: uncut hair, a
Important relics found at Mohenjo-Daro dagger, a pair of shorts, a comb, and a steel
include: bangle. Under him, the community
• Mother Goddess Idol. consolidated as a socio-religious and military
• Bronze Dancing Girl. force.
• Priest King. Reference: NCERT, Class 12th, Themes in
• Pashupati Seal. Indian History, Part II, Theme 6
• Seven stranded necklace.
• Mohenjo-Daro Ruler (Ivory), etc. 61. Ans: D
Reference: NCERT, Class 12th, Themes in Exp:
Indian History, Part I, Chapter 1. Statement 1 is correct: Samharta, was the
highest officer, in charge of tax assessment
60. Ans: D and Sannidhata was the chief custodian of the
Exp: state treasury and storehouse.
Baba Guru Nanak (169-1539) was born into a Statement 2 is correct: The Mauryan army
merchant family in a village called Nankana was well organized and it was under the control
Sahib near the river Ravi in the predominantly of Senapati. The salaries were paid in cash.
Muslim Punjab. He was married at a young age, Kautilya refers to the salaries of different ranks
but he spent most of his time among sufis and of military officers. According to Greek author
bhaktas. He also travelled widely. Pliny, the Mauryan army consisted of six lakh
Statement 1 is correct: He advocated a form infantry, thirty thousand cavalry, nine thousand
of Nirguna Bhakti. He firmly repudiated the elephants and eight thousand chariots. In
external practises of the religions he saw around addition to these four wings, there were the
him. He rejected sacrifices, ritual baths, Navy and Transport and Supply wings.

https://upscpdf.com/ https://upscpdf.com/
https://upscpdf.com/ https://upscpdf.com/

16

The administration of armed forces Statement 2 is correct: Magadha was a region


according to Megasthenes was carried out by where agriculture was especially productive.
a board of 30 officers , divided into 6 Besides, iron mines (in present-day
committees of 5 members each, each of these Jharkhand) were accessible and provided
committees was responsible for a particular resources for tools and weapons.
wing of the armed forces. Statement 3 is correct: Elephants, an
Statement 3 is correct: The Mudrarakshasa important component of the army, were found
written by Visakhadatta is a drama in Sanskrit. in forests in the region. Also, the Ganga and its
Although written during the Gupta period, it tributaries provided a means of cheap and
describes how Chandragupta with the convenient communication.
assistance of Kautilya overthrew the Reference: NCERT, Class 12th, Themes in
Nandas. It also gives a picture of the socio- Indian History I, Chapter 2
economic condition under the Mauryas.
Reference: NCERT, Class 12th, Themes in 64. Ans: A
Indian History, Part I, Theme 2 Exp:
Pair 1 is correct because Kutagarashala
62. Ans: C refers to a hut with a pointed roof where
Exp: debates were held in the Buddhist tradition
Al-Biruni was born in 973, in Khwarizm in of India. It was a place where scholars and
present day Uzbekistan. Khwarizm was an monks engaged in rigorous debates and
important centre of learning, and Al-Biruni discussions on various philosophical and
received the best education available at the religious topics. The term "kutagara" literally
time. Al-Biruni’s Kitab-ul-Hind, written in means "pointed roof," and "shala" means
Arabic, is simple and lucid. (Hence, Pair I is "hut" or "hall," emphasizing the purpose of
correct) this building as a centre for intellectual
Ibn Battuta, whose full name was Abu discourse and dialectical exchange.
Abdullah Muhammad Ibn Battuta, and who is Pair 2 is correct because both Rajasuya and
known for his work Rihla, was born in Ashvamedha were elaborate sacrifices
Morocco around 1304 AD. (Hence, Pair II is performed by kings in ancient India, including
correct) during the time of the Buddha.
François Bernier, a Frenchman, was a doctor, • Rajasuya: The rajasuya sacrifice is
philosopher, and historian. (Hence Pair III is one of the major Vedic rituals
correct) mentioned in ancient Indian texts like
Reference: NCERT, Class 12th, Themes in the Mahabharata and the
Indian History II, Chapter 1 Ramayana. It was a grand and
complex ceremony conducted by a
63. Ans: D king to symbolize a king's
Exp: ascendancy to the rank of a universal
The Mahājanapadas were sixteen kingdoms or monarch, marking his supremacy over
oligarchic republics that existed in Northern other rulers.
ancient India from the sixth to fourth centuries • Ashvamedha: The Ashvamedha
BCE. Magadha (in present-day Bihar) became sacrifice, also known as the "horse
the most powerful mahajanapada. Various sacrifice," was another significant
factors responsible were: Vedic ritual performed by kings to
Statement 1 is correct: Early Buddhist and establish their power and
Jaina writers who wrote about Magadha dominance. It involved the
attributed its power to the policies of consecration of a specially chosen
individuals: ruthlessly ambitious kings of horse, wherever the horse roamed
whom Bimbisara, Ajatasatru, and freely without being challenged was
Mahapadma Nanda are the best known, and considered to be the domain of the
their ministers, who helped implement their monarch.
policies. Pair 3 is incorrect because both Mahavira and
Buddha questioned the authority of the Vedas,

https://upscpdf.com/ https://upscpdf.com/
https://upscpdf.com/ https://upscpdf.com/

17

and played a central role in reforming the endowments for hospices and granted tax-free
religious and social order of ancient India. land (inam). The Chishtis accepted donations in
• Mahavira: Mahavira was the founder cash and kind. Rather than accumulate
of Jainism. He advocated a path of donations, they preferred to use these fully on
spiritual liberation through self- immediate requirements such as food, clothes,
discipline and the abandonment of living quarters and ritual necessities (such as
worldly attachments, in contrast to the sama).
Vedic emphasis on rituals and caste- Reference: NCERT, Class 12th, Themes in
based duties Indian History, Part II, Theme 6
• Buddha: Siddhartha Gautama, who
later became known as the Buddha, 66. Ans: A
also challenged the authority of the Exp:
Vedas and the prevailing religious 1. Mahajanapadas (about 2500 years ago):
norms of his time. He rejected the The Mahajanapadas were ancient
elaborate Vedic rituals and the caste kingdoms or territorial states that emerged
system. Instead, he taught the Four in the Indian subcontinent around the 6th
Noble Truths and the Eightfold Path century BCE. They followed the Vedic
as a means to attain enlightenment period and were a precursor to the Mauryan
and liberation from suffering. Empire.
Reference: NCERT, Class 12th, Themes in 2. Digha Nikaya, an ancient Buddhist text,
Indian History I, Theme 4 which contains some speeches of Lord
Buddha, was written down 2300 years
65. Ans: C ago.
Exp:
3. End of the Ganas or Sanghas (about 1500
Of the groups of Sufis who migrated to India in
years ago): The Mauryan and Gupta
the late twelfth century, the Chishtis were the
Empires, among others, played a role in the
most influential. This was because they adapted
decline of these republics. Around 1500
successfully to the local environment and
years ago, the last of these forms of
adopted several features of Indian devotional
decentralized governance disappeared as
traditions.
they were finally conquered by Gupta
Statement 1 is incorrect: Most Sufi lineages
rulers.
were named after a founding figure. For
Reference: NCERT, Class 6th, Our Pasts-I,
example, the Qadiri order was named after
Chapter 5
Shaikh Abd’ul Qadir Jilani. However, some,
like the Chishti order, were named after their
67. Ans: C
place of origin, in this case, the town of Chisht
Exp:
in central Afghanistan.
Statement 1 is correct: Gigantic statues of
Statement 2 is incorrect: Khwaja
Kushana rulers have been discovered within a
Muinuddin, popularly known as “Gharib
shrine at Mat near Mathura, Uttar Pradesh.
Nawaz” (comforter of the poor). The earliest
Comparable statues have also been found in an
textual references to Khwaja Muinuddin’s
Afghan shrine. These findings suggest that the
dargah date to the fourteenth century. It was
Kushanas might have viewed themselves with
evidently popular because of the austerity and
divine qualities. Several Kushana rulers also
piety of its Shaikh, the greatness of his spiritual
took on the title "devaputra," meaning "son
successors, and the patronage of royal visitors.
of god," possibly influenced by Chinese rulers
Statement 3 is incorrect: A major feature of
who referred to themselves as "sons of heaven."
the Chishti tradition was austerity, including
Statement 2 is correct: Numerous gold coins
maintaining a distance from worldly power.
were issued in the first century CE by the
However, this was by no means a situation of
Kushanas. These were virtually identical in
absolute isolation from political power. The
weight to those issued by contemporary
Sufis accepted unsolicited grants and
Roman emperors. The widespread use of gold
donations from the political elites. The Sultans
coins indicates the enormous value of the
in turn, set up charitable trusts (auqaf ) as
transactions that were taking place.

https://upscpdf.com/ https://upscpdf.com/
https://upscpdf.com/ https://upscpdf.com/

18

Reference: NCERT, Class 12th, Themes in Reference: NCERT, Class 12th, Themes in
Indian History, Part I, Theme 2 Indian History, Part II, Theme 6

68. Ans: C 70. Ans: B


Exp: Exp:
Statement 1 is correct: In 1017, when Sultan Option B is correct: During the rule of
Mahmud invaded Khwarizm, he took several Ashoka, arrangements were made for disposing
scholars and poets back to his capital, Ghazni; of affairs of the people and to receive regular
Al-Biruni was one of them. He arrived in reports about them. Pativedakas were
Ghazni as a hostage but gradually developed a appointed to report about the affairs of the
liking for the city, where he spent the last days people at all times to the Ashoka. Epigraphists
of his life. have translated the term Pativedaka as a
Statement 2 is correct: Travel literature was reporter.
already an accepted part of Arabic literature by Reference: NCERT, Class 12th, Themes in
the time Al-Biruni wrote Kitab-ul-Hind. This Indian History, Part I, Theme 2
literature dealt with lands as far apart as the
Sahara desert in the west to the River Volga in 71. Ans: A
the north. Exp:
Reference: NCERT, Class 12th, Themes in Option A is correct: Sir Alexander
Indian History II, Chapter 1 Cunningham in 1854 published Bhilsa Topes,
one of the earliest works on Sanchi. It
69. Ans: B comprised a brief historical sketch of the rise,
Exp: progress, and decline of Buddhism.
Statement 1 is incorrect: The seventh to ninth Sir Alexander Cunningham was a British Army
centuries saw the emergence of new religious Officer and Archaeologist who excavated many
movements, led by the Nayanars (saints sites in India including Sanchi and Sarnath, and
devoted to Shiva) and Alvars (saints devoted he also served as the first Director of the
to Vishnu), who came from all castes, Archaeological Survey of India. Some of his
including those considered “untouchable” publications are:
like the Pulaiyar and the Panars. 1. The Ancient Geography of India
Statement 2 is correct: They were sharply (1871)
critical of the Buddhists and Jainas and 2. The Stupa of Bharhut (1879)
preached ardent love of Shiva or Vishnu as the Reference: NCERT, Class 12th, Themes in
path to salvation. They drew upon the ideals of Indian History, Part I, Theme- 4
love and heroism as found in Sangam
literature (the earliest example of Tamil 72. Ans: A
literature, composed during the early centuries Exp:
of the Common Era) and blended them with the Option A is correct: In the thirteenth century,
values of bhakti. The Nayanars and Alvars went the Ahom state depended upon forced labour.
from place to place, composing exquisite Those forced to work for the state were called
poems in praise of the deities enshrined in the paiks. A census of the population was taken.
villages they visited, and set them to music. Each village had to send a number of paiks by
Statement 3 is correct: The importance of the rotation. People from heavily populated areas
traditions of the Alvars and Nayanars was were shifted to less populated places. Ahom
sometimes indicated by the claim that their clans were thus broken up. By the first half of
compositions were as important as the Vedas. the seventeenth century, the administration
For instance, one of the major anthologies of became quite centralised.
compositions by the Alvars, the Nalayira Reference: NCERT, Class 7th, Our Pasts II,
Divyaprabandham, was frequently described Chapter 5
as the Tamil Veda, thus claiming that the text
was as significant as the four Vedas in Sanskrit
that were cherished by the Brahmanas.

https://upscpdf.com/ https://upscpdf.com/
https://upscpdf.com/ https://upscpdf.com/

19

73. Ans: D offered to various gods by various


Exp: families of poets or sages.
Options Deccan, South India, North-East, • For the purpose of singing, the hymns
Kashmir are all correct: In the first of the Rig Veda were set to tune, and
millennium BC, the big stone boulders known this collection is known as the Sam
as megaliths (literally big stones), were Veda Samhita.
carefully arranged by people and used to mark • The Yajur Veda Samhita contains not
burial sites. only prayers but also the rituals that
The practice of erecting megaliths was have to accompany them
prevalent throughout the Deccan, South India, • The Atharva Veda Samhita contains
the north-east, and Kashmir. charms and spells to ward off evil
• In the Deccan region of India, and diseases.
megalithic cultures have been • Each Samhita has added text called
associated with the advent of the Iron Brahmanas, which have
Age. commentaries on prayers and
rituals.
• Similarly, in South India, megalithic
• Each Brahmana has an Aranyaka,
sites have been discovered in states
which has mystical ritual instructions
like Tamil Nadu, Kerala, Karnataka,
to be undertaken by forest-dwelling
and Andhra Pradesh. These sites
sages.
contain various types of megalithic
• Upanishads deal with philosophical
structures and artefacts, shedding light
inquiries.
on the customs and practices of ancient
Statement 2 is incorrect:
societies in the region.
The Atharva Veda Samhita consists of
• In the north-eastern part of India, charms and spells to ward off evil and
megalithic practices have been found diseases, and its origins are attributed to non-
among certain tribal communities. Vedic culture.
These practices often involved the use Statement 3 is correct:.
of large stones to create burial Each Vedic Samhita is accompanied by
structures and memorials. Brahmanas which are commentaries on rituals
• Even in the Kashmir region, and prayers.
megaliths have been identified. Some One of the earliest known Brahmanas is the
of these megalithic structures are Shatpatha Brahmana, which sheds light on
believed to be associated with ancient the social milieu.
burial rituals and are found in areas like Reference: NCERT, Class 6th, Our Pasts I,
the Ganderbal district. Chapter 4.
Reference: NCERT, Class 6th, Our Pasts I,
Chapter 4. 75. Ans: B
Exp:
74. Ans: B Option B is correct: Mirabai (c. fifteenth-
Exp: sixteenth centuries) is one of the best-known
Statement 1 is correct: The word ‘Veda’ is woman poet within the bhakti tradition.
derived from the root ‘vid’, which means to Biographies have been reconstructed primarily
know. In other words, the term ‘Veda’ signifies from the bhajans attributed to her, which
‘knowledge/wisdom’. The Vedic literature were transmitted orally for centuries.
consists of the four Vedic Samhitas, According to these, she was a Rajput princess
Brahmans, Aranyaks, and Upanishads; the from Merta in Marwar who was married against
main collections of Vedic hymns are called her wishes to a prince of the Sisodia clan of
Samhitas. Mewar, Rajasthan. She defied her husband and
• The Rig Veda Samhita is the earliest, did not submit to the traditional role of wife and
dating from 1500 BC to 1000 BC. The mother, instead recognising Krishna, the
Rig Veda is a collection of Prayers avatar of Vishnu, as her lover. Her in-laws
tried to poison her, but she escaped from the

https://upscpdf.com/ https://upscpdf.com/
https://upscpdf.com/ https://upscpdf.com/

20

palace to live as a wandering saint composing 77. Ans: A


songs that are characterised by intense Exp:
expressions of emotion. Ibn Battuta, a Moroccan traveller travelling
According to some traditions, her preceptor overland through Central Asia, reached Sind in
was Raidas, a leather worker. This would 1333. He had heard about Muhammad bin
indicate her defiance of the norms of caste Tughlaq, the Sultan of Delhi, and, lured by his
society. After rejecting the comforts of her reputation as a generous patron of arts and
husband’s palace, she is supposed to have letters, set off for Delhi, passing through
donned the white robes of a widow or the Multan and Uch. The Sultan was impressed
saffron robe of the renouncer. Although by his scholarship and appointed him the
Mirabai did not attract a sect or group of qazi, or judge, of Delhi. He remained in that
followers, she has been recognised as a source position for several years, until he fell out of
of inspiration for centuries. Her songs continue favour and was thrown into prison. Once the
to be sung by women and men, especially those misunderstanding between him and the Sultan
who are poor and considered “low caste” in was cleared, he was restored to imperial service
Gujarat and Rajasthan. and was ordered in 1342 to proceed to China as
Reference: NCERT, Class 12th, Themes in the Sultan’s envoy to the Mongol ruler.
Indian History, Part II, Theme 6 Reference: NCERT, Class 12th, Themes in
Indian History II, Chapter 1
76. Ans: A
Exp: 78. Ans: B
Option A is correct: During the Buddha's Exp:
lifetime, none of his teachings were recorded Statement 1 is correct: Manuscripts are
in written format. Following ancient books written by hand; the term comes
Mahaparinirvana, the first Buddhist Council from the Latin word ‘manu’ meaning hand.
was held in Rajagriha, under the patronage of These were usually written on palm leaves or
King Ajatashatru, presided over by on specially prepared bark of the birch tree,
Mahakassapa, in 483 BC. Here Consensus which grows in the Himalayas.
was arrived at on how to compile the teachings Statement 2 is incorrect: Manuscripts were
of Gautama Buddha. These compilations, written in Sanskrit, but some were also
known as Tipitaka or "three baskets," were: written in Prakrit, the language used by
1. The Vinaya Pitaka- contained ordinary people, as well as in Tamil.
guidelines for those entering the Statement 3 is correct: Manuscripts dealt
monastic order (Sangha), and is with all kinds of subjects, viz., religious
attributed to Upali.( Hence Pair 1 is beliefs and practices, lives of kings, medicine
correctly matched) and science; there were also ones dealing with
2. The Sutta Pitaka contains the poems, plays, etc.
teachings of Lord Buddha and is Reference: NCERT, Class 6th, Our Pasts I,
attributed to Ananda. ( Hence Pair 3 Chapter 1
is incorrectly matched)
3. The Abhidhamma Pitaka explored 79. Ans: B
philosophical themes and is Exp:
attributed to Sarriputta. ( Hence François Bernier, a Frenchman, was a doctor,
Pair 2 is incorrectly matched) political philosopher, and historian. Like
Each pitaka consisted of various individual many others, he came to the Mughal Empire in
texts. Subsequently, Buddhist scholars search of opportunities. He was in India for
composed commentaries on these texts. twelve years, from 1656 to 1668, and was
Reference: NCERT, Class 12th, Themes in closely associated with the Mughal court, as a
Indian History, Part I, Theme 4 physician to Prince Dara Shukoh, the eldest
son of Emperor Shah Jahan, and later as an
intellectual and scientist, with Danishmand
Khan, an Armenian noble at the Mughal court.

https://upscpdf.com/ https://upscpdf.com/
https://upscpdf.com/ https://upscpdf.com/

21

Reference: NCERT, Class 12th, Themes in north India, who embellished their
Indian History II, Chapter 1 performances with gestures and songs.
Statement 2 is correct: Kathak began evolving
80. Ans: C into a distinct mode of dance in the fifteenth and
Exp: sixteenth centuries with the spread of the bhakti
Statement 1 is correct: The Maratha empire movement. The legends of Radha-Krishna were
rose to prominence during the 18th century. The enacted in folk plays called rasa lila, which
Marathas were a Marathi-speaking warrior combined folk dance with the basic gestures of
group mostly from what is now the state of the kathak story-tellers.
Maharashtra in India. They became politically Statement 3 is correct: Under the Mughal
active under the leadership of Shivaji. He emperors and their nobles, Kathak was
carved out a stable kingdom with the performed in the court, where it acquired its
support of powerful warrior families known present features and developed into a form of
as the Deshmukhs. dance with a distinctive style. Subsequently, it
Statement 2 is correct: Between 1720 and developed in two traditions, or gharanas: one in
1761, the Maratha empire expanded. It the courts of Rajasthan (Jaipur) and the other in
gradually chipped away the authority of the Lucknow. Under the patronage of Wajid Ali
Mughal Empire. Malwa and Gujarat were Shah, the last Nawab of Awadh, it grew into a
seized from the Mughals by the 1720s. By the major art form.
1730s, the Maratha king was recognised as the Reference: NCERT, Class 7th, Our Pasts II,
overlord of the entire Deccan peninsula. He Chapter 5
possessed the right to levy chauth and
sardeshmukhi in the entire region. 83. Ans: C
Reference: NCERT,Class 7th, Our Pasts II, Exp:
Chapter 8, Chapter 10 Options Kandhar, Manshera, and Kalsi are
correct: King Asoka, the third ruler of the
81. Ans: A Indian Mauryan dynasty, has come to be
Exp: regarded as one of the most exemplary rulers in
Statement 1 is incorrect: Immediate successor Indian history. In 1837, James Prinsep
of Ajatasatru was Udayin. He laid the succeeded in deciphering an ancient inscription
foundation of the new capital at Pataliputra on a large stone pillar in Delhi belonging to
situated at the confluence of the two rivers, the Ashokan rule. Several other pillars and rock
Ganges and the Son. Later it became famous as edicts (major and minor) with similar
the imperial capital of the Mauryas. inscriptions have been found scattered in more
Statement 2 is correct: The Hathigumpha than thirty places throughout India, Nepal,
inscription of Kharavela of Kalinga refers to an Pakistan and Afghanistan.
aqueduct built by King Nanda three hundred There are about 14 major rock edicts providing
years earlier. information about Ashokan policy and his
Statement 3 is correct: Nanda dynasty, that dhamma.
ruled Magadha, in northern India, between c. These 14 major Rock edicts are found in
343 and 321 BCE. The Nanda dynasty preceded ancient sites of Kandahar (Kandhar),
the dynasty of the Mauryas. Alexander invaded Manshera,
India between 327-325 BC, during the rule of Shahbazgarhi, Kalsi, Girnar, Sopara,
Dhana Nanda. Sannati, Jaugada, Shishupalgarh, etc.
Reference: NCERT, Class 12th, Themes in The Meerut Pillar (not rock edict) is now
Indian History I, Chapter 2 situated in Delhi. It was shifted from Meerut to
Delhi by Feruz Shah and erected at a location in
82. Ans: D the northern ridge of Delhi.
Exp:
Statement 1 is correct: The term kathak is
derived from katha, a word used in Sanskrit and
other languages for story. The kathaks were
originally a caste of story-tellers in temples in

https://upscpdf.com/ https://upscpdf.com/
https://upscpdf.com/ https://upscpdf.com/

22

Statement 2 is incorrect: Indore was an


important cultural centre under the
Marathas in the eighteenth century. The
Maratha empire had an efficient
administration system, with strong
encouragement for agriculture and trade.
Indore expanded and prospered as an
important centre under the Holkar’s. The
city of Ujjain expanded under Sindhia’s
patronage.
Reference: NCERT, Class 7th, Our Pasts II,
Chapter 8, Chapter 10

86. Ans: C
Exp:
Option C is correct: In addition to territorial
expansion, Akbar implemented a policy of
assimilating chieftains into the Mughal
nobility. This strategic approach proved highly
beneficial for the Mughal Empire as it garnered
Reference: NCERT, Class 12th, Themes in the support of chieftains and their armies for
Indian History, Part I, Theme 2 further conquests. While the Sisodiya Rajputs
of Mewar initially resisted Mughal authority,
84. Ans: D their eventual defeat led to an honorable
Exp: reconciliation with the Mughals. They were
An archaeological site is a location (or graciously allowed to retain their ancestral
collection of locations) where artefacts from lands, referred to as "watan," in the form of
previous human activity have been discovered. assignments known as "watan jagir." This
These artefacts may be prehistoric, historic, or diplomatic balance of defeating opponents
modern, and they form a part of the without humiliating them allowed the Mughals
archaeological record. to extend their influence over numerous kings
Pair 1 is incorrect: Mahagara is an and chieftains across the region. Notably, many
archaeological site located in Uttar Pradesh in Rajput mansabdars were granted hereditary
the district of Prayagraj. It is an important and non-transferable territories known as
Neolithic site in India. "Watan Jagir."
Pair 2 is incorrect: Burzahom is an important Reference: NCERT, Class 7th, Our Pasts II,
archaeological site located in the Srinagar Chapter 4
district in Jammu and Kashmir. It is an
important Neolithic site. 87. Ans: B
Pair 3 is incorrect: Hallur is an important Exp:
Neolithic site located in the state of Karnataka. Statement 1 is correct: According to Jaina
Reference: NCERT, Class 6th, Our Pasts I, teachings, the cycle of birth and rebirth is
Chapter 1 shaped through karma. Asceticism and
penance are required to free oneself from the
85. Ans: B cycle of karma. This can be achieved only by
Exp: renouncing the world; therefore, monastic
Statement 1 is correct: New trade routes existence is a necessary condition of salvation.
emerged within the areas controlled by the Statement 2 is incorrect: The most important
Marathas. Burhanpur, which had earlier idea in Jainism is that the entire world is
participated in the trade between Agra and animated: even stones, rocks and water have
Surat, has now expanded its hinterland to life. Non-injury to living beings, especially to
include Poona and Nagpur in the south and humans, animals, plants and insects, is central
Lucknow and Allahabad in the east. to Jaina philosophy. In contrast the Buddhist

https://upscpdf.com/ https://upscpdf.com/
https://upscpdf.com/ https://upscpdf.com/

23

philosophy propounds : the world is Reference: NCERT, Class 12th, Themes in


transient (anicca) and constantly changing; it Indian History, Part II, Theme 8
is also soulless (anatta) as there is nothing
permanent or eternal in it. Within this transient 89. Ans: A
world, sorrow (dukkha) is intrinsic to human Exp:
existence. The Nayakas were the independent Telugu
Statement 3 is incorrect: According to warrior chiefs of the Deccan plateau who
Buddhist philosophy, it is by following the provided military and administrative
path of moderation between severe penance assistance to the kings. The state of Hyderabad
and self-indulgence that human beings can rise was constantly engaged in a struggle against the
above these worldly troubles. Jainism on the Marathas to the west and with independent
other hand postulates that asceticism and Telugu warrior chiefs (nayakas) of the
penance are required to free oneself from the plateau. The Nayakas were an important part of
cycle of karma; monastic existence hence is a the Vijayanagara Administration.
necessary condition for salvation. Reference: NCERT, Class 7th, Our Pasts II,
Reference: NCERT, Class 12th, Themes in Chapter 8, Chapter 10
Indian History, Part I, Theme 4
90. Ans: D
88. Ans: C Exp:
Explanation: Option D is correct: According to François
Pair 1 is correct: The term "pargana" is Bernier, a significant distinction between
primarily associated with the Mughal Empire Mughal India and Europe was the absence of
and other medieval Indian kingdoms. In private land ownership in the former.
medieval Indian history, a "pargana" was an Bernier staunchly advocated for the merits of
administrative territorial unit that was part private property and regarded the monarch's
of a larger administrative system of a control of land as detrimental to both the state
Mughal province. and its citizens. He believed that in the Mughal
Pair 2 is correct: Peshkash was a form of Empire, the emperor possessed all land and
tribute collected by the Mughal state. distributed it among the nobility, a system he
External forces entered the forest in different deemed as having calamitous repercussions on
ways. For instance, the state required elephants the economy and society. This perspective was
for the army. So the peshkash levied from forest not unique to Bernier but was a prevalent theme
people often included a supply of elephants. in the travel accounts of the sixteenth and
Pair 3 is correct: The village panchayat was seventeenth centuries.
an assembly of elders, usually important people Reference: NCERT, Class 12th, Themes in
of the village with hereditary rights over their Indian History, Part II, Theme 5
property. In mixed-caste villages, the panchayat
was usually a heterogeneous body. An 91. Ans: C
oligarchy, the panchayat represented various Exp:
castes and communities in the village, though Statement 1 is correct: Harappan seals usually
the village menial-cum-agricultural worker was have a line of writing, probably containing the
unlikely to be represented there. The decisions name and title of the owner. Scholars have
made by these panchayats were binding on the also suggested that the motif conveyed a
members. meaning to those who could not read. Most
The panchayat was headed by a headman inscriptions are short, with the longest
known as muqaddam or mandal. Some containing about 26 signs. Although the script
sources suggest that the headman was remains undeciphered to date, it was evidently
chosen through the consensus of the village not alphabetical.
elders, and that this choice had to be ratified by Statement 2 is incorrect: It is apparent from
the zamindar. Headmen held office as long as the inscriptions that the script was written from
they enjoyed the confidence of the village right to left, as some seals show a wider
elders, failing which they could be dismissed by spacing on the right and cramping on the left as
them.

https://upscpdf.com/ https://upscpdf.com/
https://upscpdf.com/ https://upscpdf.com/

24

if the engraver began working from the right Pair 4 is correct because Kushinagara, also
and then ran out of space. known as Kusinara or Kasia, is traditionally
Statement 3 is correct: Seals and sealings considered the place where Buddha attained
were used to facilitate long-distance Parinirvana or Nibbana, which refers to his
communication, and the sealing also conveyed final passing away and the cessation of his
the identity of the sender. earthly existence. According to Buddhist texts
Statement 4 is correct: A precise and uniform and historical accounts, Buddha passed away
system of weights and measures was used in a state of meditation in Kushinagara. After
throughout the Indus Valley Civilisation, his death, his body was cremated, and the relics
pointing towards a centralised system of were distributed to various places for
administration. veneration.
Reference: NCERT, Class 12th, Themes in Reference: NCERT, Class 12th, Themes in
Indian History, Part I, Chapter 1 Indian History I, Theme 4

92. Ans: D 93. Ans: B


Exp: Exp:
Pair 1 is correct because Lumbini is widely Statement 1 is incorrect: A tradition that
recognized as the birthplace of Siddhartha developed during the medieval period in
Gautama, who later became known as Buddha. different ways was that of miniature painting.
According to Buddhist tradition and historical Miniatures (as their very name suggests) are
records, Siddhartha Gautama was born in small-sized paintings, generally done in
Lumbini, which is located in present-day water colour on cloth or paper. The earliest
Nepal. The identification of Lumbini as miniatures were on palm leaves or wood. Some
Buddha's birthplace is supported by of the most beautiful of these, found in western
archaeological evidence, including the India, were used to illustrate Jaina texts.
discovery of an ancient pillar inscription Statement 2 is correct: The Mughal
erected by the Indian Emperor Ashoka in emperors Akbar, Jahangir and Shah Jahan
the 3rd century BCE, which confirms patronised highly skilled painters who
Lumbini as the birthplace of Buddha. primarily illustrated manuscripts containing
Pair 2 is correct because Bodh Gaya is where historical accounts and poetry. These were
Siddhartha Gautama attained generally painted in brilliant colours and
enlightenment and became Buddha. portrayed court scenes, scenes of battle or
According to Buddhist texts and historical hunting, and other aspects of social life. They
accounts, Siddhartha Gautama, after years of were often exchanged as gifts and were viewed
spiritual seeking and meditation, sat under the only by an exclusive few – the emperor and his
Bodhi tree in Bodh Gaya and achieved close associates.
enlightenment, which marked a crucial turning Reference: NCERT, Class 7th, Our Pasts II,
point in his life. This event is known as the Chapter 5
"Bodhi Tree Enlightenment" or "Bodhi Gaya
Enlightenment." 94. Ans: B
Pair 3 is correct because Sarnath holds a Exp:
special place in Buddhist history as the Options 1. Mahanavmi Dibba and 3.
location where Buddha delivered his first Audience hall is correct: Domingo Paes
sermon, known as the (around 1520 CE) visited Vijayanagara during
"Dhammacakkappavattana Sutta" or the the reign of King Krishna Deva Raya, and in
"Turning of the Wheel of Dharma." After his writings, Chronica dos reis de Bisnaga
attaining enlightenment in Bodh Gaya, Buddha (Chronicles of Vijayanagar Kings), he
travelled to Sarnath (formerly known as mentioned the House of Victory as a
Isipatana) in India, where he preached the remarkable structure within the city that
Four Noble Truths and the Eightfold Path to includes ”The Mahanavami Dibba and The
his five former companions, formally initiating Audience Hall.”. These platforms are
his teaching career. significant for their historical and cultural

https://upscpdf.com/ https://upscpdf.com/
https://upscpdf.com/ https://upscpdf.com/

25

importance, as they served as a symbol of the 3. Other finds include large letters of the
empire's power and achievements. Harappan script that were carved out of
The Mahanavami dibba: is a massive white stone and perhaps inlaid in wood.
platform rising from a base of about 11,000 sq. 4. Dholavira had a unique water harvesting
ft. to a height of 40 ft. There is evidence that it system that was used to divert the waters of
supported a wooden structure. the Manhar River towards the eastern
The audience hall: a stone stairway suggests it reservoir at Dholavira.
was a two-story structure. On the platform are Reference: NCERT, Class 6th, Our Past I,
the sockets for the 100 wooden pillars that once Chapter 3.
supported the superstructure.
96. Ans: B
Exp:
Statement 1 is correct: While François
Bernier's primary focus was on portraying the
Mughal state as authoritarian, his descriptions
occasionally allude to a more intricate social
reality. For instance, he observed that artisans
lacked incentives to enhance the quality of
their products due to state appropriation of
profits, which contributed to the decline in
manufacturing.
Statement 2 is correct: Bernier also noted the
presence of a flourishing merchant
(fig. The Mahanami Dibba)
community actively engaged in long-distance
trade.
Statement 3 is incorrect: Simultaneously, he
acknowledged significant inflows of the
world's precious metals into India, attributed
to the export of manufactured goods in
exchange for gold and silver.
Reference: NCERT, Class 12th, Themes in
Indian History, Part II, Theme 5

97. Ans: C
Exp:
Option C is correct: The term
"kutagarashala" is derived from the Sanskrit
Reference: NCERT, Class 12th, Themes in
words "kuta" (meaning 'pointed') and "shala"
Indian History, Part II, Theme 7
(meaning 'hut' or 'shelter'). It essentially
describes a shelter or hut with a distinctive
95. Ans: D
pointed roof. Debates occurred in the
Exp: kutagarashala,or within groves where
The city of Dholavira was located on Khadir traveling mendicants (a member of a
Beyt in the Rann of Kutch, where there was religious order) would rest.
fresh water and fertile soil. Some of the features In the debate, if one of the Contenders could
unique to the settlement of Dholavira were: convince the other of his perspective, then the
1. Dholavira was divided into three parts, disciples of the other contender would also start
and each part was surrounded with massive following him. This was one of the ways in
stone walls, with entrances through which support of any sect could increase or
gateways. decrease over a period of time.
2. There was also a large open area in the Reference: NCERT, Class 12th, Themes in
settlement, where public ceremonies could Indian History, Part I, Theme- 4
be held.

https://upscpdf.com/ https://upscpdf.com/
https://upscpdf.com/ https://upscpdf.com/

26

98. Ans:A
Exp:
Domingo Paes (around 1520 CE), in his
writings “Chronica dos reis de Bisnaga
(Chronicles of Vijayanagar Kings), wrote,
“What I saw (Vijayanagar) was some-what
as large as Rome and very beautiful to light.
This is the best-provided city in the
world". It reflects the city's wealth and
prosperity, architectural splendour, urban
planning, markets, and overall prosperity Pair 3 is correct: The Mahanavami Dibba is
during the Vijayanagara Empire's zenith in the associated with various ceremonial and
16th century. administrative purposes; one of the prominent
Reference: NCERT, Class 12th, Themes in rituals is the celebration of Mahanavami.
Indian History, Part II, Theme 7

99. Ans: C
Exp:

Pair 4 is correct: The Stone Chariot in Hampi


is one of the most iconic and recognised
structures of the Vijayanagara Empire inside
the Vittala Complex.
It was built in the 16th century by the orders
Pair 1 is correct: The Lotus Mahal (named by of King Krishnadevaraya, a Vijayanagara
British travellers in the nineteenth century) is ruler.
also known as the Kamal Mahal or Chitrangi Reference: NCERT, Class 12th, Themes in
Mahal. While the exact purpose of the Lotus Indian History, Part II, Theme 7
Mahal is not definitively known, as Mackenzie
wrote, it may have been a council chamber, 100. Ans: A
a place where the king met his advisers. Exp:
Option A is correct: The term which Indo-
Persian sources of the Mughal period most
frequently used to denote a peasant was
raiyat or muzarian. In addition, we also
encounter the terms kisan or asami. Sources of
the seventeenth century refer to two kinds of
peasants – khud-kashta and pahi-kashta.
Reference: NCERT, Class 7th, Our Pasts II,
Chapter 5
Pair 2 is incorrect: Hazara Rama temples:
Some historical interpretations and theories
suggest that this was probably meant to be
used only by the king and his family. Scenes
from the Ramayana sculpted on the inner walls
of the shrine.

https://upscpdf.com/ https://upscpdf.com/

You might also like